Download as docx, pdf, or txt
Download as docx, pdf, or txt
You are on page 1of 87

ENGLISH

(General Education)
EVELYN P. MAGDALENA, Ph. D
February 17, 2023

General English Examination for CBC


1. Which of the following must be avoided in an ethical communication?
a. Avoid the use of plural constructions
b. Make use of gender-neutral words
c. Glaring grammar incorrectness
d. Falsified information

2. Which should be excluded from the minutes of the meeting?


a. condemnation or honors
b. motion raised
c. number of members
d. to make the meeting peaceful

3. What refers to a document that encompasses all facets of a subject matter?


a. interview
b. letter
c. report
d. observation

4. Which of the following is an example of memorized speech?


a. impromptu speech
b. oration
c. extemporaneous speech
d. declamation

5. According to the American Psychological Association (APA), the practice of


claiming credit for the words or ideas of others is known as which of the
following?
a. plagiarism
b. paraphrasing
c. creativity
d. academic freedom

6. Globalization in the 21st century is inevitable because of the growing


interconnectedness across the countries. Which should be a primary
consideration in cross-cultural communication?
a. uses wheelchair
b. respect and tolerance of others culture
c. confined to a wheelchair
d. awareness of others' culture
7. Which of the following is an example of a scenario that needs a memo?
a. minutes of the meeting
b. announcement regarding the changes in the company policy
c. memorandum
d. termination of an employee

8. Which serves as an essential element in responding to a letter of complaint?


a. consistency
b. clarification
c. creativity
d. empathy

9. What register should be used in writing business letters?


a. non-formal
b. creativity
c. informal
d. formal

10. What refers to the written records of a meeting's proceedings?


a. records
b. minutes
c. memorandum
d. notice

11. Robert's Rule of Order (2005) suggested 11 elements to be contained in a


meeting's minutes. Which of the following comes after the kind of meeting
(regular, special, annual, etc.)?
a. name of the company
b. name of the members
c. number of the participants
d. date, location, and time of the meeting's beginning

12. What refers to an official note constructed to inform employees about certain
announcements or information?
a. minutes of the meeting
b. memorandum
c. approval
d. notice of meetings

13. What term is used to refer to communication that occurs between people from
the same ethnic group?
a. cultural communication
b. international communication
c. intracultural communication
d. interracial communication
14. What term is used to refer to communication that takes place between people of
different races?
a. cultural diversity
b. international communication
c. cultural relativism
d. interracial communication

15. Dianne, a newly licensed psychologist, would like to apply for a job. Which
document should she prepare to highlight her qualifications for employment?
a. application email
b. progress report
c. diploma
d. resume

16. Which type of public speaking brings out the most natural information to say at
the moment due to minimal to no preparation time given?
a. oratorical
b. memorized speech
c. impromptu speech
d. valedictory address

17. Which of the following is included in the communication's environment or setting?


a. Feedback
b. Mood
c. Source
d. Channel

18. When a public speaker examines the audience's interests, age and maturity
before drafting a speech, he is adhering to which of the following?
a. Consideration
b. Cultural Sensitivity
c. Concreteness
d. Creativity

19. "Only three out of ten people in the world are able to eat a complete meal in a
day". This message is effectively evoking which of the following principles of
effective communication?
a. Concreteness
b. Cultural Sensitivity
c. Consideration
d. Creativity

20. What refers to the content being sent in a communications?


a. Message
b. Channel
c. Mood
d. Source

21. Which feature should be present in a resume to achieve completeness of


qualifying information?
a. consistency
b. comprehensiveness and relevance
c. focus and visual impact
d. creativeness

22. Which of the following acts as a cultural barrier for effective communication to
take place in a global setting?
a. cultural relativism
b. cultural diversity
c. international communication
d. interracial communication

23. Which type of resume stresses key information by means of bold or italic
typeface?
a. application email
b. print resume
c. scannable
d. application letter

24. What can cause communication to be ineffective?


a. Channel
b. Context
c. Interference
d. Cultural Barrie

25. Communication usually begins when the message is crafted by whom?


a. Context
b. Receiver
c. Sender
d. Feedback

26. What effective use of communication can teachers utilize to explain a certain
learning content creatively to learners?

a. texting
b. storytelling
c. letter
d. writing an email

27. What should be done to an application letter for greater relevance to the
company an individual is applying for?
a. inclusion of property rights information
b. inclusion of all personal information
c. inclusion of all real property information
d. inclusion of significant personal information

28. When a person wishes to know more about any information, he should write
which type of letter?
a. love letter
b. inquiry letter
c. excuse letter
d. expository letter

29. According to the American Psychological Association, writers must use the
words, "man" for boys and "woman" for girls who are both over 18 years old.
This is one way to appropriately prevent, which of the following form of
discrimination?
a. Ageism
b. Make use of gender-neutral words
c. Avoid the use of plural constructions
d. Sexism

30. Which is achieved by a public speaker who uses eye contact?


a. authenticity
b. creativity
c. correctness
d. clarity

31. Which of the following can be used to effectively analyze an organization's status
to achieve improvement?
a. Board Resolution
b. SWOT
c. SQ3R
d. Memorandum

32. What must be avoided in writing business letters?


a. formal
b. non-formal
c. Brevity
d. business jargons

33. Where can the expression of expectation and gratitude for the recipient be
found?
a. body of the letter
b. first paragraph
c. final paragraph
d. complimentary close
34. What refers to the purposive verbal communication where the aim is to seek
answers to queries on certain topics?
a. text
b. interview
c. letter
d. email

35. In 2009, Michael Osborn identified nine principles for effective communication.
Which of these is achieved when jargon is avoided?
a. Consideration
b. Cultural Sensitivity
c. Correctness
d. Clarity

36. Therese, a deaf student, was not able to give an answer to her teacher's
question during the recitation because she lacks the ability to hear verbal
instruction, which kind of inference was illustrated?
a. Mechanical Barrier
b. Psychological Barrier
c. Emotional
d. Physical Barrier

37. In a debate, Joe said, "I strongly disagree with a same-sex marriage because
allowing it makes an imbalance on the number of straight women and men,
which then can cause lesser families to be established". Which type of fallacy
was used?
a. memorized speech
b. slippery slope
c. red herring
d. impromptu speech

38. Which type of purposive communication is utilized by a criminal lawyer who aims
to convince the judge by proving his client is innocent?
a. informative communication
b. argumentative communication
c. persuasive communication
d. international communication

39. Which type of report does Anne need to create when she needs to present the
status of the company's project?
a. progress report
b. feasibility report
c. news report
d. Newsletter
40. When asked to deliver an extemporaneous speech, Mark experienced stage
fright, which hindered his ability to speak smoothly, causing the audience to not
fully understand him. What communication barrier is present?
a. Psychological Barrier
b. Physical Barrier
c. Linguistic Barrier
d. Cultural Barrier

41. What does the letter "O" represents in SWOT analysis?


a. Organization
b. Optimal
c. Online
d. Opportunity

42. Sexism is a form of prejudice and discrimination based on an individual's gender.


How can one avoid gender-biased communication?
a. Make use of gender-neutral words
b. Ageism
c. Avoid the use of plural constructions
d. Sexism

43. Communication has numerous purposes. Hence, the intention of the sender
greatly affects the choice of words in a message. Which type of purposeful
communication is exhibited by Mario, a fish vendor, when he encourages market
goers to buy from him?
a. international communication
b. informative communication
c. argumentative communication
d. persuasive communication

44. In communicating with people with disabilities, which of the following is an


acceptable term to address them?
a. respect and tolerance of others culture
b. uses wheelchair
c. confined to a wheelchair
d. awareness of others' culture

45. In writing an academic or business letter, what information should be contained


in the first paragraph?
a. background or purpose of writing
b. salutation
c. message
d. specific information or content

46. How does one prevent himself from plagiarizing?


a. creativity
b. paraphrasing
c. academic freedom
d. plagiarism

47. According to Business Insider Australia, some questions are illegal to be asked
to an applicant during an interview. Which of the following is an example of this?
a. your birthdate
b. your previous employment experiences
c. your full name
d. your height and weight

48. In government or legal meetings, why must an outsider facilitate the minutes of
the meeting?
a. to avoid biases
b. to have better result
c. to gain full attention throughout the meeting
d. to make the meeting peaceful

49. What causes glaring and grammatical and typographical errors in an application
letter?
a. comprehensiveness and relevance
b. extreme nervousness
c. lack of proofreading
d. creativeness

50. What can confirm the recipient's understanding of the message in a


communication?
a. Message
b. Context
c. Receiver
d. Feedback
GENERAL EDUCATION (FILIPINO)

Naikokonsidera sa pagpapakahulugan ng mga diskurso na may kaugnayan sa


gulang, katayuan at hangarin ng kausap. Anong anyo ng konteksto ng diskurso
ito?
A. Historikal
B. Sosyal 
C. Kultural
D. Sikolohikal
 
May kaugnayan ito sa tradisyon, gawi, paniniwala, at prinsipyo ng kausap. Anong
anyo ng konteksto ng diskurso ito?
A. Historikal
B. Sosyal
C. Sikolohikal
D. Kultural
 
 
Ang mga kasapi ay nabibilang sa isang particular na kasarian tulad ng usapang
lalake at usapang babae. Anong konteksto ito?
A. Interpersonal
B. Pangmasa
C. Interkultural
D. Pangkasarian

 
Ang SONA ay isang halimbawa ng anong konteksto?
A. Interkultural
B. Pangmasa 
C. Interpersonal
D. Pangkasarian
 
Sa batayang uri ito, ang akda ay naglalayong mapatunayan ang katotohanan ng
ipinahahayag at ipatanggap sa bumabasa ang katotohanang ito.
A. Eskspositori
B. Deskriptib
C. Naratib
D. Argumentatib
 
 
Ang pagbibigay ng biswal na konsepto sa pamamagitan ng paggamit ng mga
angkop at wastong mga salita upang malinaw na maipakita ang katotohanan ng
tao, bagay,pook,  at maging ang mga pangyayari.
A. Eskspositori
B. Naratib
C. Argumentatib
D. Deskriptib
 
 
Mga kwentong sinasalaysay, karaniwang mayroon itong tagapagsalaysay tulad ng
mga maikling kwento, nobela, at iba pang panitikan.
A. Naratib
B. Deskriptib
C. Argumentatib
D. Eskspositori
 
Anumang teksto na nagpapaliwanag o naglalahad ng mga kaalaman hinggil sa
anumang paksang pasaklaw ng kaalaman ng tao.
A. Deskriptib
B. Naratib
C. Argumentatib
D. Eskspositori
 
Ang mga sumusunod na pahayag ay dapat isaalang-alang upang maging mabisa
ang komunikasyon, maliban sa?
A. Norms
B. Setting
C. Gender
D. Participants

Ang discursus ay nangangahulugang __________ at __________.


A. argument at pagsasalita
B. argument at kumbersasyon 
C. argument at pagpapaliwanag
D. argument at paglilinaw
Panuto: Piliin ang mga wastong salita upang mabuo ang diwa ng mga
pangungusap.
 
Nag-aaral ang __________ at __________ na bata.
A. mahuhusay - masipag
B. mahusay - masipag
C. mahusay - masisipag
D. mahuhusay - masisipag

Nagkamit ng gantimpala ang ____________ na atleta.


A. matalinong malakas
B. matalino at malalakas
C. matalino na malakas 
D. matalino at malakas
 
__________ na sa kani-kanilang bahay ang mga batang nagsipaglaro.
A. nagsiuwi 
B. nagsisiuwi
C. magsiuwi
D. magsisiuwi
 
Marami sa mga taga-Morong ang nagkantahan at __________ sa plaza noong
nakaraang pista.
A. nagsayaw
B. nagsayawan
C. sumayaw
D. sumasayaw
 
Makita lang kita, ako’y ______________.
A. sasaya at sisigla 
B. matutuwang masigla
C. liligaya at masigla
D. tatawang masigla

Piliin sa mga sumusunod na pahayag ang may wastong ayos ng pangungusap.


A. Nakita niya ang magandang larawan ni Lia.
B. Nakita niya ang larawan ni Lia na maganda.
C. Nakita niya si Lia at ang larawan niyang maganda.
D. Nakita niya ang larawan na maganda ni Lia.
 
Piliin sa mga sumusunod na pahayag ang may wastong ayos ng pangungusap.
A. Si Lyca ay maganda at tunay na mayumi.
B. Si Lyca ay tunay na maganda at mayumi. 
C. Si Lyca ay tunay na maganda at tunay na mayumi.
D. Si Lyca ay tunay na magandang mayumi
 
Piliin sa mga sumusunod na pahayag ang may wastong ayos ng pangungusap.
A. Ang ating mga ninuno ay sumasamba sa kalikasan at naniwala sa
makapangyarihang diyos na tinatawag nilang Bathala.
B. Ang ating mga ninuno ay sumamba sa kalikasan at naniniwala sa
makapangyarihang diyos na tinatawag nilang Bathala.
C. Ang ating mga ninuno ay sumamba sa kalikasan at naniwala sa
makapangyarihang diyos na tinatawag nilang Bathala.
D. Ang ating mga ninuno ay sumamba sa kalikasan at naniwala sa
makapangyarihang diyos na tinatawag nilang Bathala.
 
 
Piliin sa mga sumusunod na pahayag ang may wastong ayos ng pangungusap.
A. Si Arken ay nagigising, bumangon, at dumungaw sa bintana.
B. Si Arken ay nagising, bumangon, at dumudungaw sa bintana.
C. Si Arken ay nagising, bumangon, at dumungaw sa bintana.
D. Si Arken ay nagising, bumabangon, at dumungaw sa bintana.
Piliin sa mga sumusunod na pahayag ang may wastong ayos ng pangungusap.
A. Sina Ben at Tricia ay sumasakay ng dyip at bumaba sa harap ng Simbahan ng
Quiapo
B. Sina Ben at Tricia ay sumakay ng dyip at bumaba sa harap ng Simbahan ng
Quiapo 
C. Sina Ben at Tricia ay sumakay ng dyip at pababa sa harap ng Simbahan ng
Quiapo
D. Sina Ben at Tricia ay sumakay ng dyip at bumababa sa harap ng Simbahan ng
Quiapo
Panuto: Basahing mabuti ang mga pahayag. Tukuyin at piliin ang salitang nagpamali
sa pangungusap.
 
Maluwang ang bibig nang mama. 
A. Maluwang
B. bibig
C. nang
D. mama
 
May dumi sa pagmumukha si George. 
A. dumi
B. George
C. pagmumukha
D. may
 
Ginanahan sa paglamon ang mga anak ni Lia. 
A. Anak
B. Ginanahan
C. Paglamon
D. Lia
 
Maarte siya sa pagkain kaya hindi siya tumataba. 
A. Hindi
B. Pagkain
C. Maarte
D. tumataba
 
Malaswa ang damit ng babae.
A. Ang
B. Damit
C. Malaswa
D. Babae

Panuto: Tukuyin kung tama o mali ang mga ipinahahayag ng mga


pangungungusap. 
 
Tama o Mali: Matalim ang kanilang kutsilyo tulad ng kutsilyo natin
A. Mali kasi tamang-pang-ugnay sa pagsasama-sama o paghihiwalay ng mga ideya
sa loob ng pangungusap ang ginamit
B. Mali kasi hindi magkatugma ang pangngalang itinutulad at ang bagay na
pinagtutularan gayundin ang paggamit ng wasto at angkop na panghalip panao
C. Tama kasi magkatugma ang pangngalang itinutulad at ang bagay na
pinagtutularan gayundin ang paggamit ng wasto at angkop na panghalip
panao
D. Tama kasi tamang-pang-ugnay sa pagsasama-sama o paghihiwalay ng mga
ideya sa loob ng pangungusap ang ginamit
 
Tama o Mali: Nagsisiuwi na sa kani-kanilang bahay ang mga batang magsipaglaro.
A. Mali kasi hindi nagkakaisa ang aspekto ng pandiwang gagamitin
B. Tama kasi ang pang-uri ay nakasunod sa pangngalan
C. Mali kasi hindi nakasunod ang pang-uri sa pangngalan
D. Tama kasi nagkakaisa ang aspekto ng pandiwang gagamitin
 
Tama o Mali: Ang kanyang bunganga ay punong-puno habang nagsasalita
A. Ang pangungusap ay mali dahil ang pahayag niya ay nagpapakita ng negatibong
salita
B. Ang pangungusap ay mali dahil dapat bibig ang kanyang ginamit at hindi
bunganga.
C. Lahat ng nabanggit ay tama.

 
Tama o Mali: Pupunta kami kila Sandra bukas ng hapon
A. Ang dapat gamitin ay “nang” hindi “ng”
B. Ang pangungusap ay mali dahil wala namang salitang kila
C. Ang pangungusap ay tama kase why not.
D. Wala sa nabanggit.

Tama o Mali: Tatakot din siyang pumunta sa malayong lugar


A. Tama ang pangungusap at paggamit ng din dahil ito ay nagtatapos sa
katinig
B. Tama ang pangungusap at paggamit ng din dahil ito ay nagtatapos sa patinig
C. Mali, dahil rin ang dapat gamitin kapag nagtatapos sa patinig
D. Mali, dahil rin ang dapat gamitin kapag nagtatapos sa katinig

Makikipag-usap ako ____ Bea at Jude


A. kila
B. kina
 
Tawa ______ tawa ang bata habang naglalaro sa parke
A. nang
B. ng

_____ Binangonan ako!


A. Tiga-
B. Taga-

_____ prutas siyang dala sa amin


A. Mayroon
B. May
 
Si Anya ay nagpapanggap na ___________ sa kanilang bayan
A. May
B. Mayroon
 
 
Ang kaibigan ko ay isa lamang maralita.
A. Mabait
B. Mangmang
C. Maliit na tao
D. Mahirap

Si Nena ay inaruga  ng kanyang lola mula pa noong siya’y naulila.     


A. Pinabayaan
B. Pinamigay
C. Kinuha
D. inalagaan
 
 
Nangangamba  ka ba na hindi ka niya pagbigyan?      
A. nababanas
B. Natatakot
C. nasisiyahan
D. Naiinis
 
Ang pagpunta sa ibang bansa ay hindi gawang biro.
A. Maayos
B. mahirap
C. madali
D. Masayang gawin
 
Nakaririmarim ang nangyari sakuna sa dagat.     
A. nakakainis
B. Nakapangingilabot.
C. nakalulungkot
D. Nakatatakot
SCIENCE
(General Education)
MR. JEANNE PAUL S. RAYMUNDO
February 17, 2023

General Science Review


1. What is the first element in the periodic table?
A. Helium
B. Hydrogen
C. Oxygen
D. Carbon
2. Tropical disturbances are classified as tropical depressions, tropical storms, or
typhoons. This classification is based on the ________.
A. Amount of rainfall
B. Strength of the accompanying winds
C. Amount of rainfall and strength of accompanying winds
D. Origin of formation

3. When a plant cell is placed in a hypertonic solution, the plant cell will:
A. expand
B. rupture
C. shrink
D. have no effect on the plant cell.

4. Which of the following organisms is included in the Kingdom Plantae?


A. Magnoliidae
B. Fungi
C. Protozoa
D. Prokaryotes

5. Which of the following forms of energy travels in waves?


A. Chemical energy
B. Sound energy
C. Light energy
D. Electrical energy

6. A soda-acid type of fire extinguisher is recommended for putting out fires that
involve burning _______.
A. insulation on wires
B. vegetable oil
C. dry chemicals
D. panted chemicals

7. To achieve ecological balance, which of the following sets of components should


an aquarium contain.
A. Water, fish
B. Water, snail, fish
C. Water, plants, fish
D. Water, plants, snail, fish

8. The first Filipino who was declared a natural scientist and who contributed much
in discovering local plants that can be used as a medicine is _____.
A. Dr. Pacifico Marcos
B. Dr. Henry Moseley
C. Dr. Eliseo Kintanar
D. Dr. Alfredo Santos

9. Metabolism and combustion are chemically similar. They both result in the
oxidation of some compounds. If completely oxidated, which of the following
foodstuffs would yield the MOST thermal energy?
A. A cup of ice cream
B. A cup of sugar
C. A cup of milk
D. A cup of flour

10. All of the following are reasons why terraces are built in Mountain Province,
EXCEPT:
A. to accommodate excess run of water
B. to provide space for planting rice
C. to make the place beautiful
D. to provide steps for the natives to go uphill

11. Sulfuric acid is not used in


A. Making Fertilizers
B. Detergents
C. All of the above
D. None of the above

12. Solar eclipse occurs when ________.


A. The Moon is between the Earth and the Sun.
B. The Earth is between the Moon and the Sun
C. The Sun is between the Earth and the Moon.
D. The Earth is behind the Sun.

13. During processing and cooking which part of food is mostly destroyed?
A. Fats
B. Proteins
C. Vitamins
D. Carbohydrates
14. What will most likely happen when a species cannot adapt to the changes in the
environment?
A. It will be transformed into another form
B. It will become extinct
C. It will be isolated
D. It will grow old

15. A British physician and bacteriologist, Sir Ronald Ross was awarded the Nobel
Prize in medicine for his research in the _____.
A. liquid fuel rocket
B. prevention of hepatitis
C. transmission of malaria
D. control of cholera

16. The following means help in controlling floods, EXCEPT:


A. Reforestation
B. Deforestation
C. Crop rotation
D. Construction of floodways

17. If a doctor describes a patient as dehydrated, he means that the patient ______.
A. has a contagious disease
B. needs insulin
C. needs oxygen
D. has lost a great deal of water

18. Petroleum is formed by the _______.


A. decay of microorganisms under heat and pressure
B. heat from a radioactive substance found in the mantle
C. volcanic eruptions that burn living matters
D. steam trapped underneath a geologic column

19. What phase of the Moon is observed when it cannot be seen at all because it
passes directly between the Earth and the Sun?
A. New Moon
B. First Quarter
C. Last Quarter
D. Full Moon

20. Astronauts strap themselves to a wall or bunk when they sleep. This is because
during a change of shuttle velocity an unstrapped sleeping astronaut would likely
slump into the shuttle wall and be injured. These astronauts have to be strapped
down because?
A. They are unable to sleep while floating in free space.
B. Gravity is weak to have them remain in bed.
C. Their body functions have to be constantly monitored.
D. Their bodies have inertia.

21. Which of the following parts of the circulatory system carries digested fats away
from the small intestines?
A. Pancreatic duct
B. Arterial capillaries
C. Pulmonary artery
D. Lacteals

22. Two glass tumblers that are stuck one inside the other may be loosened by ____.
A. pouring cold water on the outside tumbler after filling the inside tumbler
with equally cold water.
B. pouring hot water on the outside tumbler after filling the inside tumbler with
equally hot water.
C. pouring cold water on the outside tumbler after filling the inside tumbler
with hot water.
D. pouring hot water on the outside tumbler after filling the inside
tumbler with cold water.

23. Radium was discovered by:


A. Albert Einstein
B. Pierre and Marie Currie
C. Joseph John Thompson
D. Albert van Haller

24. When water evaporates, it changes into which of the following states?
A. Solid
B. Matter
C. Gaseous
D. Liquid

25. Which of the following causes the seasons?


A. The rotation of the Earth around the Sun.
B. The rotation of the Sun around the Earth
C. The distance of the Earth around the Sun
D. The tilting of the Earth on its axis.

26. Spiders can be distinguished from insects by the fact that spiders have _______.
A. hard outer covering
B. four pairs of legs
C. large abdomens
D. biting mouthparts

27. The two chemical factors that can cause decomposition are:
A. death and cellular respiration
B. autolysis and photosynthesis
C. autolysis and putrefaction
D. putrefaction and respiration

28. Which part of the brain is responsible for intelligence, memory, and learned
behavior?
A. Cerebellum
B. Cerebrum
C. Medulla
D. Thalamus

29. A lot of malnourished children in Africa are found to have poor teeth and
eyesight. This is believed to be caused by a deficiency in ______.
A. Vitamin A and D
B. Iodine
C. Vitamin C
D. Iron

30. Acid becomes colorless when added with


A. Litmus indicator
B. Methyl Orange Indicator
C. Universal Indicator
D. Phenolphthalein Indicator

31. Vapor, a gas, is observed to leave from an ice, which is solid. Which process is
being observed?
A. Evaporation
B. Condensation
C. Sublimation
D. Decomposition

32. Which of the following statements is NOT TRUE about comets?


A. Comets are mostly frozen gases and cosmic dust.
B. Comets glow by their own lights
C. Comets generally have elongated elliptical orbits.
D. Comets may go around the sun in any direction.

33. When lit, the energy of the alcohol in a lamp is?


A. lost while heating is done.
B. transformed from chemical to mechanical energy.
C. transformed from chemical energy to heat energy.
D. completely destroyed.

34. What is the color of the hottest star in the Universe?


A. red
B. blue
C. yellow
D. orange

35. In what state are most matters in the universe?


A. Gas
B. Liquid
C. Solid
D. Plasma

36. Which of the following statements best explain why copper is the metal most
widely used in electrical wiring?
A. It is the best conductor of electricity
B. It is cheaper than aluminum.
C. It is a better conductor than aluminum and cheaper than silver.
D. It has high resistance to electricity.

37. A plesiosaur fossil was dug by an archaeologist in the Sahara Desert. In which
particular rock might it most likely be encased?
A. Metamorphic rocks
B. Igneous rocks
C. Sedimentary rocks
D. Ocean floor

38. Two items of similar shapes and weights are dropped simultaneously in a
building. Which of the following statements is correct about falling objects?
A. The shape of the object has a minor effect on the falling rate.
B. The weight of the object has no effect on the falling rate.
C. The distance of the fall has an effect on the falling rate.
D. The resistance of the air has no effect on the falling rate.

39. A cold-blooded animal is one that _____.


A. lacks red corpuscles
B. lacks white corpuscles
C. has a variable body temperature
D. has a fixed body temperature

40. How will genetic drift affect the population of rabbits that are living in a stable
habitat with no changes in the environment?
A. It will increase the population of rabbits.
B. It will decrease the genetic variation of the rabbits.
C. It will develop the dominance and superior traits of the rabbits.
D. It will enhance the appearance of new traits of the rabbits.

41. In countries where medical skills and public understanding of health and disease
are widespread, which of the following changes are evident?
I. Accidental deaths have decreased.
II. The communicable disease death rate has decreased.
III. Infant mortality rate has decreased.
IV. More people live a full life.
V. People who live long enough succumb to degenerative disease.

A. I, II, III
B. II, III, IV, V
C. I, II, III, IV, V
D. I, II, III, IV

42. The Philippines lies in the region where many volcanoes are active. This region
is known as:
A. Wheel of fire
B. Ring of fire
C. Volcanic Rim
D. Archipelagic Fault Line

43. A scientist uses a thermometer to measure ____.


A. energy
B. heat
C. pressure
D. temperature

44. An atom or a group of atoms that has an excess/deficiency of electrons is called


______.
A. neutron
B. proton
C. ion
D. molecules

45. The history of the earth has been divided by scientists into eras. Which of the
following shows the arrangement of these eras into a sequence from the oldest
to the most recent?
A. Precambrian, Mesozoic, Cenozoic, Paleozoic.
B. Paleozoic, Cenozoic, Precambrian, Mesozoic.
C. Cenozoic, Mesozoic, Paleozoic, Precambrian.
D. Precambrian, Paleozoic, Mesozoic, Cenozoic.

46. Which of the following statement(s) is/are TRUE?


I. Water in a cup and water in a pot at 100 degrees Celsius have molecules
of the same degree of activity.
II. It takes more burning of fuel to produce a pot of boiling water than to
produce a cup full of water.
III. The unit used in measuring the quantity of heat is calorie.
IV. A pot full of water has a greater number of active molecules than a cup full
of water.
A. II and IV only
B. All of these statements
C. None of these statements
D. I and III only

47. Farmers add commercial fertilizers to the soil to ______.


A. Speed up the decay of bacteria
B. Add mineral content
C. Activate the growth of earthworms
D. Enhance the growth of the roots

48. Which of the following is closest to normal body temperature?


A. 32 degrees Fahrenheit
B. 50 degrees Celsius
C. 212 degrees Fahrenheit
D. 37 degrees Celsius

49. Thyroid can be destroyed through:


A. radioactive zinc
B. radioactive iodine
C. radioactive carbon
D. radioactive iodine

50. A tin can that is hammered and flattened is an example of _________.


A. mechanical change
B. physical change
C. nuclear change
D. chemical change
SOCIAL SCIENCE / ARTS
(General Education)
PROF. LEONCIO C. CATOLOS
March 3, 2023

CBC Work Exercise in Social Science (GE)


1. The 1987 Philippine Constitution was crafted out of the experience under an
authoritarian and dictatorial regime. A section of the constitution says the
Philippines is “a democratic and republican state”. Which article of the 1987
Philippine Constitution contains this provision?
A. Citizenship
B. Declaration of Principles and State Policies
C. Executive Department
D. National Territory

2. Arts and the Humanities are related to Philosophy, because artists


________________.
I. are passionate about expressing cause and advocacies through arts
II. conceive their works initially from the mind.
III. creates art pieces from illusion
IV. produces art with a set of principles and abstract meanings.

A. I and II
B. III and IV
C. I and III
D. II and IV

3. This nationalist labor leader during the American regime was also a National
Artist for Literature.
A. Amado V. Hernandez
B. Ang Kiukok
C. Lino Brocka
D. NVM Gonzales

4. What statement best defines Art?


A. Art is the expression of skills and crafts.
B. Art is the harmony of man and nature.
C. Art is the production of things of beauty.
D. Art shows differences in man’s beliefs and principles.

5. Some artists become critics and moralists by commenting on the virtues and
views of the world. This shows what function of art?
A. Economic
B. Historical
C. Political
D. Social
6. The term humanities generally refer to art, literature, music, architecture, dance
and the theatre- areas which emphasizes human
A. beliefs
B. creativity
C. knowledge
D. traditions

7. What is a traditional example of the performing arts?


A. Flash mob during a rally
B. Musicale shown in school auditorium
C. Rap played on YouTube
D. Sand painting shown on live TV

8. If a tourist visits the province of Pampanga, what item would they normally bring
home?
A. Buko pie
B. Wood products
C. Pina cloth
D. Silver jewellery

9. “Art’s for art’s sake” means art is created for _______________.


A. audience enjoyment and critic’s judgment
B. economic purpose and sold
C. public admiration
D. the artist’s own satisfaction

10. Multinational companies (MNCs) perform roles to support nations regardless of


their religion, culture, boundaries and politics in response to globalization. What
describes MNCs?
A. Business organizations that hires cheap labor
B. Businesses that produce a wide variety of products.
C. Companies that operate in two or more countries.
D. Companies that combine to dominate the market.

11. Mekong River flows through the countries of Myanmar, Laos, Thailand,
Cambodia and finally flowing out of the coast of ______________.
A. Burma
B. Malaysia
C. Singapore
D. Vietnam

12. It is a measure of the market value of the final goods and services produced by
national citizens of a country in a particular time.
A. National Income
B. GDP
C. GNP
D. Profit

13. A common characteristic of civil society organizations is_____________.


A. commercial purpose
B. political interest
C. Profit-oriented
D. Voluntary in nature

14. The 1987 constitution of the Philippines states that religious instruction may be
given to public schools provided there shall be no additional expenses to the
Government. Which of the following is NOT among the additional requirements of
the 1987 Constitution?
A. The religion teachers shall be designated by the head of the religion to
which the child belongs.
B. The adviser handling the class must be present during religion class.
C. There must be a written request by the parents or guardians.
D. The sessions shall be within the regular class hours.

15. The National Artists award was sponsored and administered by the NCCA and
the CCP. Another agency under the NCCA is _____________.
A. Movie and Television Review Classification Board
B. National Commission on Indigenous People
C. National Museum
D. Optical Media Board

16. Which of the following professions or jobs would generally be involved in a civil
society organization?
I. Farm-owner
II. Human rights lawyer
III. Labor leader
IV. Medical doctor
V. University teacher

A. I only
B. II and III
C. IV and V
D. V only

17. Foreign influence on the Filipino culture produce a different way of thinking. The
idea that what is foreign is the best and that one’s life style, products or ideas are
inferior to them is called ____________.
A. ethnocentrism
B. internationalism
C. philosophy
D. xenocentrism
18. A Spanish governor-general introduced a new calendar to the Philippine islands.
What calendar is presently being used to measure time in days and months?
A. Chinese
B. Gregorian
C. Julian
D. Lunar

19. Early Filipinos wear tattoos not only as fashion but also as a sign of __________.
A. bravery
B. marital status
C. race
D. taste in art

20. History that presents past events in the villages, towns, cities and provinces.
A. Herstory
B. International history
C. Local history
D. National history

21. Indian culture influences Philippine culture through the Sanskrit language. An
example of a Tagalog word derived from Sanskrit is____________.
A. bathala
B. bintana
C. mangkok
D. silya

22. Journals, clothes and other personal belongings of Rizal historical artifacts. One
way of determining the age of a centuries-old artifact is by ____________.
A. cryptography
B. DNA
C. Geology
D. Radiocarbon dating

23. The Cold War which divided Europe into western and eastern blocs sparked
rivalry between US and the USSR and their allies. It begun ____________.
A. after the 1st World War
B. after the 2nd World War
C. Before the 1st World War
D. Before the 2nd World War

24. One Chinese influence to Philippine culture like the words ditse and sanse is
shown in the area of ___________.
A. dress
B. family relations
C. food
D. religion

25. Arrange the following events or ideas on controversies in Philippine history


according to chronological order.
I. Cavite Mutiny
II. “Cry of Balintawak or Pugadlawin”
III. Rizal’s Retraction
IV. The First Mass in the Philippines
V. Theories on the Origin of the Filipino Race
VI. Propagation of Islamic Faith

A. I, III, V, VI, IV, II


B. IV, V, VI, I, III, II
C. V, IV, VI, I, III, II
D. V, VI, IV, I, III, II

26. National Artist for Visual Arts who was a resident of Rizal.
A. Napoleon Abueva
B. Fernando Amorsolo
C. Lucio San Pedro
D. Vincent Manansala

27. Who among these artists was awarded the honor of being National Artist in the
field of Literature?
A. Botong Francisco
B. Fernando Amorsolo
C. Carlos P. Romulo
D. Nick Joaquin

28. Guillermo Tolentino was declared a National Artist in the field of sculpture. What
prominent landmark was his work known displayed in?
A. Bonifacio monument
B. EDSA People Power monument
C. Ninoy Aquino monument in Makati
D. Rizal monument in Luneta

29. A YouTube video produced by NHCP entitled “Proclamation 1081: Martial Law”
depicting dictatorial powers and abuses of democratic and human rights rights
is ____________.
A. reliable because NHCP based the video about martial law from history
textbooks
B. reliable because the NHCP is an authority in matters relating to
Philippine history
C. not reliable since there is no YouTube yet at the time of about Martial Law
D. not reliable since Martial Law happened many years ago
30. The barangay, the earliest Filipino’s political unit, is now also the lowest local
government unit. It is an influence handed down by the_________.
A. Aetas
B. Chinese
C. Hindus
D. Malayo-Indonesian

31. Early Filipinos generally believes in the belief that life or spirit exists in the natural
environment.
A. Ancestor worship
B. Animism
C. Atheism
D. Paganism

32. Arrange the following historic events according to chronological order:


I. Birth of Christianity
II. Philippine Revolution
III. Renaissance period
IV. The two World Wars

A. I, II, III and IV


B. I, III, II and IV
C. III, II I and IV
D. IV I, II and III

33. Museums, shrines, markers, monuments, cultural indigenous performances,


practices, religious rites, and rituals are________________.
A. historical artifacts
B. primary sources
C. repositories of history
D. sources of history

34. What phrase best describes Globalization?


A. Change in ways of social transactions and relations
B. Cooperation and competition in trade and commerce
C. Fast, far and wide inter-regional flow of goods and ideas
D. Networks and interaction of social, political and economic power

35. Which of the following is the best source of the history involving the life of Jose
Rizal? It is _______________.
A. his clothes
B. his letters
C. his photos
D. textbook about his life
36. In the book, Martial Law-Looking Back 15 by foremost historian Ambeth Ocampo,
he advised to stop describing the whitewash of the Marcos dictatorship and the
martial law years as “historical revisionism.” By historical revision he meant
____________.
A. checking fake news
B. correcting what is wrong or false
C. lies and half-truths
D. social media trolling

37. What pen-name or alias did Jose Rizal used when he wrote for Diariong
Tagalog?
A. Kalipulako
B. Laong Laan
C. Plaridel
D. Tikbalang

38. Although a leading member of the Reformation Movement, Rizal did not agree on
the objectives of the Katipunan to launch revolution against Spaniards. What was
his main reason?
A. Filipinos were already contented under Spanish rule
B. Filipinos were coward during that time
C. The natives might take allegiance to the Spaniards
D. The people are not ready for revolution and resources are
insufficient

39. A periodical/newspaper which served as the organ of the Propaganda Movement


was __________.
A. Diariong Tagalog
B. Doctrina Cristiana
C. El Renacimiento
D. La Solidaridad

40. One of the greatest works of Rizal was his annotation in London of this book by
Spanish historian Antonio de Morga.
A. Mi Ultimo Adios
B. Sobre la Indolencia de los Filipinos
C. Sucesos de las Islas Filipinas
D. Un Recuerdo a mi Pueblo

41. The teaching of Jose Rizal’s life, works and writing is mandated by Rizal Law or
RA 1425. The law was authored by a Philippine senator and
nationalist____________.
A. Alejandro Roces
B. Claro M. Recto
C. Jose W. Diokno
D. Lorenzo Tanada
42. Jose Rizal is the Philippine’s official national hero. Is the statement right or
wrong?
A. Right. It is officially declared by law
B. Right. There is no need for a law to make it official.
C. Wrong. There is a law that says no one is recognized as a national hero.
D. Wrong. There is no law that makes it official.

43. Common Filipino names are of Spanish origin as a result of a Spanish Governor-
general decree in the early 1800’s.The Spanish governor general who ordered
that all Filipinos adopt a Spanish name was _____________.
A. Antonio Pigafeta
B. Francis B. Harrison
C. Narciso Z. Claveria
D. Ramon E Blanco

44. In Rizal’s letter to the women of Malolos in Bulacan, "Sa Kababaihan ng Malolos"
he praised them for their particular concern for the right to_________.
A. Education
B. Health
C. Justice
D. Nutrition

45. Arrange the following writing of Rizal, according to chronological order:


I. Noli Me Tangere
II. Mi Ultimo Adios
III. The Philippines- A Century Hence
IV. To the Filipino Youth

A. I, III, IV, II
B. II, III, I, IV
C. III, II, IV, I
D. IV, I, III, II

46. Rizal, including the Propagandists, wanted to attain political reforms in the
Philippines through free expression of ideas. What important event contributed to
the introduction of new ideas from Europe?
A. the Emancipation of Slaves
B. the Industrial revolution
C. the Opening of Suez Canal
D. the Treaty of Paris

47. Who helped finance the publication of Rizal’s Noli Me Tangere?


A. Ferdinand Blumentritt
B. Maximo Viola
C. Pio Valenzuela
D. Rufino Collantes

48. Which of the works of Jose Rizal was inspired by the martyrdom of the three
Filipino priests in Cavite?
A. El Filibusterismo
B. Noli Me Tangere
C. Mi Ultimo Adios
D. Sucesos de las Islas Filipinas

49. The closest neighboring country of the Philippines in the western side of the
South East Asian region is __________.
A. Burma
B. China
C. Laos
D. Vietnam

50. Goal number Two of Sustainable Development Goals (MDGs) aims to


____________.
A. end hunger and improve nutrition and promote sustainable
agriculture
B. ensure inclusive and equitable education for all
C. strengthen the means of implementation and revitalize the Global
Partnership for Development
D. promote sustained, inclusive and sustainable economic growth,
employment and decent work
ED 2: THE TEACHING PROFESSION
(Professional Education)
DR. DENNIS NINO ANCHETA
March 3, 2023

Mock Test - The Teaching Profession


1. What appointment can be given to Teacher A who possesses the minimum
qualifications but lacks the appropriate civil service eligibility?
a. Contractual basis
b. Provisional
c. Permanent
d. Substitute

2. How do orientation sessions help the new teacher?


I. Be attuned to the school environment
II. Learn guidelines to follow
III. Develop good relations

a. II and III
b. I and II
c. I only
d. I, II, and III

3. Which of the following provisions under the Magna Carta for Public School
Teachers will most likely promote teachers’ welfare and defend their interests?
a. Be promoted in rank and salary
b. Regulate their social involvement
c. Undergo and participate in professional development
d. Establish, join and maintain professional and self-regulating
organizations

4. Who among the following characterizes a professional teacher?


a. An education graduate who received honors
b. A teacher who has taught for at least six years
c. A teacher who has attended national seminars on teaching
d. A teacher who qualifies for a permanent position under R.A. 4670
(Magna Carta for Public School Teacher)

5. Teacher B has been in active service for 10 years when he decided to pursue
higher studies. Under R.A. 4670, what kind of leave of absence can s/he avail
of?
a. Indefinite leave
b. Study leave
c. Scholarship leave
d. Vacation leave
6. Teacher D is assigned in a rural area; Teacher E in a depressed community;
Teacher F in a hazardous area; and Teacher G in a place where standard of
living is high. Who is entitled to hardship allowance?
a. Teacher D
b. Teacher E
c. Teacher F
d. Teacher G

7. A school personnel can avail of free legal service under certain circumstances.
Principal I was accused of maligning her neighbor. Is Principal I entitled to the
said service?
a. Yes, she should defend herself.
b. No, if funds are not available
c. Yes, it might bring some disagreements in school
d. No, the case is not related to her professional duties

8. Teacher K teaches in a public school in her locality. Due to teacher shortage, her
classroom teaching starts from 6am and ends at 3pm. Is the assignment given to
her just?
a. Yes, the situation demands that she render longer teaching hours
b. Yes, as long as she signs a conforme letter to that effect.
c. No, rendering longer teaching hours would make the teacher tired and
exhausted
d. No, Magna Carta for Public School Teachers states that in the exigencies
of service, any teacher may be required to render more than six hours and
not more than eight hours of actual classroom teaching a day

9. Teacher O tutors her students, who have difficulty coping with Math, after class
hours. Is her act ethical?
● Yes, provided she receives just compensation.
● Yes, provided she does not require a fee from the parent
● No, that is unfair to other students.
● No, she should be free after her official time

10. What should a teacher do when s/he falls in love with his/her student?
a. Court the student at home
b. Propose and marry the student
c. Wait till the student is no longer under his/her tutelage
d. Act normally as if nothing happens

11. When a Principal starts to exercise his/her powers over making and promoting
students, is his/her action acceptable?
a. Yes, when the teacher cannot make decision on time
b. Yes, when there is abuse of judgement on the part of the teacher
c. No, teachers are more knowledgeable of their student’s performance
d. No, grading and promoting students are exclusive functions of
teachers

12. Teacher S, a Science teacher has been reported to the office of sexual
harassment by one of her students. What should the principal do first?
a. Ask the teacher to surrender to the police
b. Tell the teacher to stop reporting to school
c. Answer and submit a report to the division office
d. Create a committee to investigate the accusation

13. Dr. Velasco, a schools’ division superintendent acted on the complaint filed by a
group of parents against the alleged misconduct of a particular teacher. She
issued a memorandum requiring her to take a leave of absence for a week while
the complaint is being heard yet. Was the action of the superintendent legal?
a. Yes, because she is the superintendent
b. No, because the complaint has not been heard yet
c. Yes, the superintendent has disciplinary authority over teachers
d. No, the superintendent has no disciplinary authority over teachers

14. What norm of conduct is manifested by being loyal to the republic and to the
Filipino people?
a. Professionalism
b. Responsiveness to the public
c. Nationalism and Patriotism
d. Honesty

15. Which of the following is true about the teacher as a person under the Code of
Ethics for Professional Teachers?
a. Live with dignity at all times wherever s/he is
b. Serve as a model worthy of emulation
c. Place premium upon self-respect and self-discipline
d. All of the above.
ED 3 – THE TEACHER AND THE COMMUNITY, SCHOOL CULTURE AND
ORGANIZATIONAL LEADERSHIP COMPETENCY BUILDING COURSE SY. 2021-
2022

1. It is known as “Enhanced Basic Education Act of 2013”


A. R.A. 10533
B. R.A. 9155
C. R.A. 4670
D. R.A. 10157
 
2. Which of the following statements best describe teacher leaders?*
A. Not only are these leaders concerned and involved in the process; they are also
focused on helping every member of the group succeed as well.
B. They step outside their classroom doors and accept the challenges to
improve their practice through working with colleagues, school administration
and professional staff—as well as students and their families .
C. They communicate with their staff and together set clear goals related to student
achievement.
D. All of the choices
 
3. Public Schools District Supervisors are the highest educational leader in ________
level.*
A. Regional
B. Division
C. Distict
D. School

4. How many principles or components that School-Based Management comprises?*


A. 1
B. 2
C. 3
D. 4

5. E-SIP stands for ____________________________________.*


A. Enhanced School Improvement Plan
B. Enhanced School Institutional Plan
C. Enhanced School Intervention Plan
D. Enhanced School International Plan
  
6. “The learning system collaboratively developed and continuously improved, anchored
on the community and learner’s contexts and aspirations”. Which SBM principle best
describes the statement?*
A. Leadership and Governance
B. Curriculum and Instruction
C. Accountability & Continuous Improvement
D. Management of Resources
 
7. It refers to a group of people who "share a concern or a passion for something they
do and learn how to do it better as they interact regularly".*
A. Community of Practice
B. Community Involvement
C. Community Partnership
D. Community Engagement

8. AIP stands for _________________________________.*


A. Annual Implementation Plan
B. Annual Improvement Plan
C. Annual Intervention Plan
D. Annual Interface Plan
  
9. The highest official in the DepEd Central Office.*
A. Secretary of Education
B. Regional Director
C. Undersecretaries
D. Commissioners
 
10. Which of the following is the correct sequence of organizational levels in the
Department of Education?*
A. National Regional Division District School
B. National Division Regional District School
C. National Regional District Division School
D. National Division District Regional School
 
11. Who is the current Secretary of the Department of Education?*
A. Sarah Duterte
B. Annalyn Sevilla
C. Diosdado San Antonio
D. Francis Cesar Bringas
  
12. School-Based Management is also considered as shared governance.*
A. True
B. Not true
C. Either A or B
D. Cannot be determined
 
13. The following are the key processes in the crafting of School Improvement Plan.
Which DOES NOT belong?*
A. Plan
B. Respond
C. Act
D. Assess
14. CPD means _______________________________. This includes attending
training, seminars, post-graduate education and the like.*
A. Continuing Program Development
B. Continuing Professional Development
C. Continuing Project Development
D. Continuing Partnership Development
 
 
15. In school-based management, which of the following corresponds to Principle I?*
A. Accountability & Continuous Improvement
B. Management of Resources
C. Leadership and Governance
D. Curriculum and Instruction
  
16. It refers to a ‘web of relationship’ and these relations are fundamental for
understanding human behavior and different institutions.*
A. Society
B. Community
C. Educational Bodies
D. Cultural diversity

17. Which the following term that best described as a major aspect of development of
any modern society and most significant indicator of overall development of any region.*
A. Globalization
B. Community
C. Education
D. Competence
  
18. This refers to an exchange between two or more individuals and is a building block
of society. Social interaction can be studied between groups of two (dyads), three
(triads) or larger social groups.*
A. Community
B. School Practice
C. Social Interaction
D. Culture
 
19. The following statements describe school culture. Which is NOT ?*
A. School culture can be defined as the guiding beliefs and values evident in the way a
school operates.
B. School culture can be used to encompass all the attitudes, expected behaviors and
values
C. School culture involves attitudes, behaviours and values that impact how the school
operates.
D. School culture is generally seen as the foundation of society which brings
economic wealth, social prosperity and political stability.
 
20. It is a model of school leadership in which a principal works alongside teachers to
A. Teacher Leadership
B. Evolutionary Leadership
C. Transformational Leadership
D. Instructional Leadership 
 
21. Who is the highest official in the Schools Division Office?*
A. Public Schools District Supervisor
B. Education Program Supervisor
C. Schools Division Superintendent
D. Education Program Specialists
 
22. Who is the immediate supervisor in school?*
A. Public Schools District Supervisor
B. Officer of the day
C. School Head
D. Administrative Officer
 
23. Which of the following is national level in nature?*
A. Regional Office
B. Central Office
C. District Office
D. Division Office
  
24. They are the leaders who are the professionals who carry through with this process
to lead change in their schools for the benefit of all students.*
A. Instructional leaders
B. Transformational leaders
C. Teacher leaders
D. All of the choices
  
25. They leaders who are generally energetic, enthusiastic, and passionate.*
A. Teacher leaders
B. Instructional leaders
C. Transformational leaders
D. All of the choices
  
26. Which of the following corresponds to Principle 2 in SBM?*
A. Management of Resources
B. Leadership and Governance
C. Curriculum and Instruction
D. Accountability & Continuous Improvement
  
27.  Principle 3 in the school-based management focuses on
________________________.*
A. Management of Resources
B.Leadership and Governance
C. Curriculum and Instruction
D. Accountability & Continuous Improvement

28.  In school-based management, Principle 4 focuses on


________________________.*
A. Leadership and Governance
B. Curriculum and Instruction
C. Accountability & Continuous Improvement
D. Management of Resources
 
29. Who is the highest official in DepEd-Regional Office?*
A. Regional Director
B. Assistant Regional Director
C. Schools Division Superintendent
D. Asst. Schools Division Superintendent
 
30. Brigada Eskwela and Adopt-a-School program focuses on the different aspects
EXCEPT one;*
A. Curriculum Development
B. Community Partnership
C. Community Engagement
D. Building Connections

31. Through the involvement of teachers, parents, and other community stakeholders in
these key decisions, SBM can create more effective learning environments for children.*
A. True
B. Not True
C. Either true or not
D. Insufficient information
 
 
32. It is a road map of the school that lays down specific interventions that a school will
undertake within a period of three consecutive school years.*
A. AIP
B. E-SIP
C. SBM
D. LAC
 
33. It is a school plan which outlines exactly how you will be reaching your goals for the
year, and who in your team is responsible for that outcomes.
A. SBM
B. SLAC
C. AIP
D. E-SIP
34. It is an annual activity led by DepEd where everyone in the school community –
including the private sector – work together to prepare schools for the opening of the
classes*
A. Adopt-a-School Program
B. Learning Action Cell
C. Graduation and Recognition Rites
D. Brigada Eskwela
 
35. Which of the following statements best described annual implementation plan of
school?*
A. Collectively and judiciously mobilized and managed with transparency,
effectiveness and efficiency.
B. Create more effective learning environments for children.
C. Monitors expected and actual performance of students.
D. It links strategic goals and objectives to tactical goals and objectives.
 
36. This style of leadership connotes the phrase "Do as I say."*
A. Authoritative
B. Autocratic
C. Pacesetting
D. Democratic
 
37.  In this style of leadership, the leader gives people a little direction to help them tap
into their ability to achieve all that they're capable of.*
A. Affiliative
B. Coaching
C. Democratic
D. Laissez-Faire
 
38. It refers to all training, certification and education that a worker needs to succeed in
his or her career. It's no secret that different jobs require different skills.*
A. Education
B. Professional Development
C. Teaching-Learning
D. Workshop
 
39. What does PPST stand for?*
A. Philippines Professional Styles for Teachers
B. Philippine Performance Standards for Teachers
C. Professional Philippine Standards for Teachers
D. Philippine Professional Standards for Teachers
 
40. In which DepEd Order is PPST stipulated?*
A. DO No. 42, s. 2017
B. DO No. 42, s. 2018
C. DO No. 42,s. 2019
D. DO No. 42,s. 2020

41. Teacher leaders step outside their classroom doors and accept the challenges to
improve their practice through working with colleagues, school administration and
professional staff—as well as students and their families .*
A. True
B. Not True
C. Maybe
D. Either B or C
  
42. Through professional development, workers can learn many skills to become better,
more efficient workers.*
A. True
B. Not True
C. Maybe
D. Either B or C
 
43. LAC stands for ________________________________.*
A. Leadership Action Cell
B. Learning Action Cell
C. Levelling Action Cell
D. Least Action Cell

44. It refers to a strategy to improve education by transferring significant decision-


making authority from the Central Office to individual schools.*
A. School Learning Action Cell
B. School-Based Management
C. Strategic Leadership
D. School Improvement Plan
  
45. It provides principals, teachers, students, and parents greater control over the
education process by giving them responsibility for decisions about the budget,
personnel, and the curriculum.*
A. School Learning Action Cell
B. Professional Development Plan
C. School-Based Management
D. School Improvement Plan

46. SBM stands for ____________________________________.*


A. School-Based Management
B. School-Based Managerial
C. School-Based Monitoring B.
D. School-Based Mandatory
 
47. Public Schools District Supervisors are the highest educational leader in
___________ level.*
A. Regional
B. Division
C. District
D. School

48. “A network of leadership that provides the vision and direction to the education
system making it relevant and responsive to the contexts of diverse communities”.
Which SBM principle best described the statement?*
Leadership and Governance
Curriculum and Instruction
Accountability & Continuous Improvement
Management of Resources
 
49. “Resources are collectively and judiciously mobilized and managed with
transparency, effectiveness and efficiency”. Which SBM principle best described the
statement?*
A. Leadership and Governance
B. Curriculum and Instruction
C. Accountability & Continuous Improvement
D. Management of Resources
 
 50. “A clear, transparent, inclusive and responsive accountability system is in place,
collaboratively developed by community stakeholders, which monitors expected and
actual performance, continually addresses the gaps, and ensures a venue for feedback
and redress”. Which SBM principle best described the statement?*
A. Leadership and Governance
B. Curriculum and Instruction
C. Accountability & Continuous Improvement
D. Management of Resources
ED 4

1. Task:  Brushing Teeth 1. Pick up the tooth brush 2. Wet the brush3. Take the cap off
the tube 4. Put paste on the brush 5. Brush the outside of the bottom row of teeth 6.
Brush the outside of the top row of teeth 7. Brush the biting surface of the top row of
teeth. This is an example of _______________.
a. Task Analysis
b. Backward Chaining
c. Video Based-intervention
d. Forward Chaining
2. What disorder manifest problem on movement and posture that results from damage
to the areas of the brain that control motor movement?
a. Intellectual Disability
b. Cerebral Palsy
c. Stereotypic movement disorder
d. Developmental Coordination Disorder

3. Children with ___________ has moderate to severe disability and may also have
problems with self-care.
a. Learning disability
b. Autism
c. ADHD
d. Down Syndrome
 
4. Work that deprives children of their childhood, their potential and their dignity, and
that is harmful to physical and mental development is the description of ________.
a. Displaced Children
b. Child Labor
c. Indigenous People
d. Abused Children
5. What legal mandate states the equal rights and privileges of PWDs on employment,
education, health, telecommunications, auxiliary social services, accessibility, political,
and civil rights?
a. RA 7277 (1992)
b. BP 344 (1983)
c. RA 10754 (2016)
d. RA 10366 (2013)
6. What educational setting assess, instructs and evaluate students individually and
intentionally?
a. Mainstreaming
b. General Education
c. Special Education
d. Inclusive Education
 
7. A systematic approach to planning curriculum and instruction for academically
diverse learners is called __________.
a. General Adaptation
b. Universal Design for Learning
c. Differentiated Instruction 
d. Individualized Education Program
8. A child who has been forced to flee his or her country because of persecution, war or
violence is called _________.
a. Child Labor
b. Abused Children
c. Displaced Children
d. Indigenous People
9. Which of the following disabilities manifest speech and language difficulties that affect
social development and communication skills of the child ?
a. Hearing Difficulty
b. Intellectual Disability
c. Visual Difficulty
d. Autism
10. David is a 4-yr.-old who has a form of cerebral palsy (CP) called spastic diplegia.
How will you classify David’s condition?
a. Disability
b. Ability
c. Impairment
d. Handicap
11. Which mandate proposed the policy on special education?
a. World Education Forum Framework for Action and the Millennium Summit
b. UN Convention on the Rights of Persons with Disabilities in 2006
c. Education 2030 Framework for Action
d. The Salamanca Statement and Framework for Action on Special Education
(1994)

12. A child who does not experience or follow the normal course of development and
reaches a milestone earlier or later than other children in his/her age is called
_______________ development.
a. Typical
b. Abnormal
c. Atypical 
d. Disable
 
13. What disability has normal intellectual functioning but has significant gap or
discrepancy between assumed potential and actual achievement?
a. Learning Disability
b. Gifted
c. Autism
d. Intellectual Disability
14. Which model views disability as blessing or curse?
a. Moral/ Religious Model
b. Social Model
c. Right Based Model
d. Biomedical Model
15. Students who have difficulty understanding the lessons and  unable to do homework
because their parents are uneducated that unable to teach them. They are also lacking
of school supplies because there is not enough money to buy them. Sometimes they
are excluded and discriminated. They are considered as ____________.
a. Learning Disable
b. Intellectual Disable
c. Poor People
d. Marginalized Learners
 
16. Julia likes to read, she is also excited to learn new concepts. She often gives
uncommon responses to common questions but  sometimes she easily get bored in the
class. These are some indications of __________.
a. Giftedness
b. ADHD
c. Talented
d. Autism
17. What disorder is characterized by repetitive, seemingly drive, and nonfunctional
motor behavior?
a. Intellectual Disability
b. Stereotypic movement disorder
c. Visual Difficulty
d. Developmental Coordination Disorder
18. Dina is having difficulty in reading and spelling. She often mixed up sentences and
write different spelling of the same word. This is an indication that Dina has
____________.
a. Visual Impairment
b. Dysgraphia
c. Dyslexia
d. Dyscalculia
19. What will be the next procedure after assessment results are processed?
a. Placement
b. Pre-Referral
c. Referral
d. Special Education
20. Liza used to speak loudly.  Sometimes she does not respond when she being
called.  She always watches the face of whoever she talks to, sometimes she requests
to repeat the information again and again.  Liza might be suffering from
_______________.
a. Learning Disability
b. Visual Difficulty
c. Hearing Difficulty
d. Intellectual Disability
21. What educational setting includes all the learning and supplementary services inside
the general education classroom?
a. General Education
b. Inclusive Education
c. Special Education
d. Mainstreaming
22. A child who is having difficulty in catching  an object, using scissors or cutlery,
handwriting, riding a bike, or participating in sports might be suffering from __________.
a. Developmental Coordination Disorder
b. Stereotypic movement disorder
c. Intellectual Disability
d. Visual Difficulty
23. Raymond is having difficulty putting words into sentence sometimes he find it
difficult to find the right word to say, he always say “uh” whenever he cant say what he
wanted to say.  Raymond may be suffering from _______________.
a. Autism
b. Receptive Language Disorder
c. Expressive language Disorder
d. Hearing Difficulty
24. What response accommodation is appropriate for a child with dysgraphia?
a. Graphic organizers
b. Concrete models and manipulatives
c. Scribe to record dictated responses 
d. Word processor with spelling and grammar check
25. Which of the following is NOT a sign of Attention Deficit Hyperactivity Disorder?
a. Easily distracted and sidetracked
b. Difficulty in fine and motor skills
c. Does not follow instruction and fails to finish schoolwork
d. Does not pay attention to details and works
26. Teachers are supposed to present information and materials in many different ways,
taking into consideration that some students may have not fully absorbed the lesson the
first time it was presented. She must utilize ________.
a. Differentiated Instruction
b. Individualized Education Program
c. Universal Design for Learning
d. Modification
27. What is the least restrictive environment for a child with additional needs?
a. General education classroom
b. residential facility
c. Special Education Class
d. home- or hospital-based programs
28. What is the placement of children with multiple disabilities, often of the severe kind?
a. Restrictive classroom
b. home- or hospital-based programs 
c. Special Education with supplementary instruction and services
d. Special Education Classroom
29. Ricky has a poor academic performance in reading and writing. He is  also clumsy,
and usually bump into objects which causes him to fall down. Ricky might be suffering
from___________.

a. Intellectual Disability
b. Visual Difficulty
c. Hearing Difficulty
d. Learning Disability
30. Which disability model believes that PWDs are seen as disabled not because they
are deficient but because people “insists” they are deficient and disadvantaged?
a. Social Model
b. Biomedical Model
c. Moral/ Religious Model
d. Right Based Model
31. In special education, teachers, administrators, school staff, other workers, parents
and families, community, and government are consider as __________.

a. Stakeholders 
b. Professionals
c. Proponents
d. Persons in-charge
32. What disability manifest intellectual deficits ?
a. Gifted
b. Intellectual Disability 
c. Learning Disability
d. Autism
33. Grasping the meaning of quantities like biggest vs. smallest, understanding that the
numeral 5 is the same as the word five, counting money or making change, and
estimating time are manifestations of having ______________.
a. Dyscalculia
b. Dyslexia
c. Visual Impairment
d. Dysgraphia
34. What educational setting integrate the child with additional needs into a general
education setting?
a. Special Education
b. Inclusive Education
c. Mainstreaming
d. General Education
35. Which of the following has symptoms like stuttering, difficulty in articulation and
producing sounds?
a. Hearing Disability
b. Learning Disability
c. Language disorder
d. Speech Impairment

36. If a child has visual difficulty, what accommodation in presentation should be used?
a. Advance organizer or story guide
b. Copy of directions
c. Note-taking support
d. Minimize visual distraction

37. It is one of the accommodations being used in Inclusive Education, wherein the
design of instructional materials and activities are modified to make the content
information accessible to all children.
a. Individualized Education Program
b. Universal Design for Learning
c. Differentiated Instruction
d. Modification
38. Task:  Put on a T-shirt. 1.  Pull shirt over head, 2.  Push right arm up through right
sleeve 3.  Push left arm up through left sleeve 4. Pull shirt down to waist.This is an
example of ________.
a. Task Analysis
b. Forward Chaining 
c. Video Based-intervention
d. Backward Chaining
 
39. Kyrie finds it difficult to sustain attention in his tasks and easily distracted.  Kyrie
shows the sign of having __________.

a. Intellectual disability
b. Learning disability
c. ADHD 
d. Autism
40. A state or quality of being different or varied; which may relate to their race,
ethnicity, gender, sexual orientation, language, culture, religion, mental and physical
ability, class, and immigration status is called ____________.
a. Mainstreaming
b. Inclusive
c. Special Education
d. Diversity
ED 5&6
ASSESSMENT IN LEARNING

1. Teacher Alma is confused about the similarity or difference between


measurement and evaluation, so she asked Teacher Beth about it. Teacher Beth
explained to Teacher Alma that measurement and evaluation are the same or
similar concepts. Is Teacher Beth correct?
A. Yes, as measurement and evaluation both involve the collection of
information on learners' learning.
B. No, as measurement and evaluation are related but distinct processes in
assessing learners' learning.
C. Yes, as measurement and evaluation are part of the assessment process.
D. No, as measurement is performed after evaluation.
2. A product learning target is best matched with which of the following sample
assessment tasks?
A. Demonstrating how to operate a printer in front of the class.
B. Writing a reflection paper on the role of culture in learning.
C. Answering a sentence-completion paper-and-pencil test.
D. Drafting a script for the class stage play.

3. What type of assessment requires learners to demonstrate a task?


A. paper and pencil B. performance-based
C. norm-referenced D. criterion-referenced

4. Why do teachers need to monitor the progress of learners' performance in


educational tests?
A. Majority of learners need to pass the subject.
B. Several failures reflect on the quality of assessment used.
C. Every learner's learning is an accountability of the teacher.
D. Many parents will complain if learners fail the subject.

5. Which statement is WRONG based on the Revised Bloom's Taxonomy of


Cognitive Learning?
A. When one interprets a given picture, he/she performs at "understanding
level" in the taxonomy of cognitive behavior.
B. The objective is considered testing application level if it requires use of
knowledge and skills in a real-world setting.
C. If the objective begins with the verb "critique", then the cognitive behavior is
considered under creating level.
D. When one compares two concepts, then he/she functions cognitively under
analyzing level.

6. What is the number one disadvantage or weakness of fill-in-blank completion test


items?
A. Ease in constructing test items
B. Difficulty to assess higher levels of learning
C. Scoring can be tedious and time-consuming
D. Possibility that there is more than one answer

7. What strategy can be done to increase the internal consistency of a test?


A. Provide a strict test proctor.
B. Increase the number of items.
C. Inform the learners to answer honestly.
D. Control the testing administration conditions.

8. Refer to the box-and-whisker plot below that shows the results of an aptitude test
given to selected college entrants.

Based on the Score


Distribution of Samples of Girls
and Boys, which are the
correct median scores for girls and boys, respectively?
A. 77, 72.5
B. 75, 70
C. 76, 72.5
D. 77,73

9. In a final examination, Mr. Gabriel noticed that the distribution of test scores of
his learners had an extremely large standard deviation. What does this
observation indicate?
A. The number of learners who took the test is large.
B. The test measures a wide range of knowledge.
C. The learners performed very well in the test.
D. The learners' scores vary quite a bit.

10. What is the main reason why teachers need to assign or give grades to learners'
test results or performance?
A. To give feedback on specific topic/s learners have mastered and areas that
they need to improve on.
B. To guide school administrators on what to prioritize for their next planning
and decision-making.
C. To advice learners that they need to drop the subject as they will likely to fail.
D. To communicate to the parents their child's class standing.
11. Which model of alternative assessment focuses on determining the effects of
Start Here instruction rather than determining if students have achieved the
intended outcomes?
A. Authentic Assessment
B. Developmental Assessment
C. Emergent Assessment
D. Performance Assessment

12. Teacher K is confused about the nature of alternative assessment of learning. A


colleague, Teacher M, explained to Teacher K that alternative assessment refers
to the use of nontraditional assessment methods like portfolio assessment. Is
Teacher M correct?
A. Yes, as portfolio assessment is a nontraditional assessment method.
B. No, as portfolio assessment is a traditional assessment method.
C. Yes, as alternative assessment means using portfolio in assessment.
D. No, as alternative assessment and nontraditional assessment are different.

13. A product learning target is BEST matched with which of the following sample
assessment tasks?
A. Measuring height using a tape measure
B. Explaining the difference between vectors and matrices
A. C Demonstrating commitment to completing the group project
C. Writing an essay on the impact of social media in people's social values

14. Asa culminating activity of the Physical Education subject of Grade 6 students
the P.E. teachers chose to stage a field demonstration of the different folk
dances. Each Grade 6 class has to select a type of folk dance (eg, Tinikling,
Pandanggo sa llaw, Itik-itik) to perform. The teachers then decided to develop a
rubric to guide them in assessing the students' performance. What type of criteria
should be used for this purpose?
A. Content Criteria, because it will evaluate the degree of a student's knowledge
and understanding of how ethnic or folk dance should be done.
B. Impact Criteria, because it will evaluate the overall effects of the students'
dance performance on the audience.
C. Process Criteria, because it will evaluate the proficiency or skill level of
performance of students in folk dancing
D. Quality Criteria, because it will evaluate the quality of their dance
performance.

15. Every student in Mr. Ramos' Grade 9 class seems to bring unique problems into
the classroom. More than 40 percent of the students migrated from a politically
stricken nearby region. Each meeting seems to be in chaos. Many students
appear restless during class sessions. Others who are quiet have eyes drooping
to sleep. Many seek permission to go out every now and then for a drink or visit
the rest room or clinic because of a headache. Mr. Ramos, as a class adviser,
has been reminded of significant absenteeism in this class. What affective trait
most likely contributes to the classroom incidents?
A. Political belief
B. Self-concept
C. Anxiety
D. Motivation

16. You are going to assess the writing skills of the students using portfolio
assessment. Which of the following will you ask your students to do?
A. Let them do a reflective journal for their lessons discussed in the class
B. Prepare a quality container for their written outputs in class.
C. Collect all their works in the class and compile them.
D. Select the evidence of their learning.

17. Mr. Bautista is teaching Arts subject in one public elementary school. He plans to
require students to come up with a project that will demonstrate their learning in
this subject. He also needs to develop a valid assessment tool since this
requirement will greatly affect the students' final grade. He thought of coming up
with a rubric. What is the first step that he needs to do?
A. A Determine and describe performance task to be evaluated and its quality
attributes.
B. Determine the benchmarks and point values to denote the levels of
performance.
C. Develop the criteria or dimensions for assessment.
D. Write the benchmark or performance descriptors for each criterion and point
value.

18. What response format is used to describe the object or behavior by making use
of two opposite adjectives?
A. Likert Scale
B. Verbal Frequency Scale
C. Graphic Scale
D. Semantic Differential Scale

19. Why is feedback helping students improve their performance?


A. It points out the part of the performance where there is a mistake,
B. It tells specific ways to change the work and make it better.
C. It makes the teacher think whether to make the learners resubmit.
D. It helps the teacher provide high grade in the tasks required.

20. Why should feedback be given immediately?


A. So that students can revise their answers immediately.
B. Because the teacher needs to maximize the classroom time.
C. Several students need to be accommodated with limited time.
D. So that learners will realize their mistakes.
ED 7

1. Every time a new unit is taken up, the teacher presents students with an advanced
organizer. Which principle does the teacher apply?
A. Arrange for appropriate practice.
B. Provide correct responses on the first trial.
C. Organize material into appropriate learning units.
D. Assist students to learn communication skills.
2. It is sound to encourage students to define terms in their own words because
________
A. defining the terms in their own words help them memorize the definition faster.
B. this is one opportunity to brush up with their English.
C. they ought to connect the terms they learn with other terms.
D. students remember information better when they mentally process it in
some way.
 
3. What learning process involves the application of previously acquired learning to new
or different situations?
A. retention
B. conditioning
C. motivation
D. transfer
 
4. Student Basil had fun at school yesterday. Today, he is eager to go to class. What
theory explains this?
A. Classical conditioning
B. Associative learning
C. Operant conditioning
 D. Affective filter hypothesis
 
5. What practice is exemplified when a principal emphasizes the need for a clean and
orderly classroom as well as clean and beautiful surroundings to contribute to effective
teaching and learning?
A. Provide an atmosphere conducive to learning.
B. Provide adequate physical facilities.
 C. Utilize educational technology.
D. Establish rapport between teachers and pupils.
 
6. Which comment helps create a conducive learning environment?
A. "How many problems can you get right?"
B. "Today I want to see how much you can learn."
C. "Here are five difficult problems. It's all right if you can just solve three."
D. "Here are five difficult problems. It's all right if you can just solve three."
7.  As student B was solving a math problem she detected an error and recovered from
that error by making a quick fix. Which ability did student B displayed?
A. Metacognition
B. Intuition
C. Imagination
D. Creative thinking
 
8. Which practice is based on Thorndike’s connectionism?
A. Make use of drill and rewards for students to learn
B. Use collaborative learning often
C. Make use of mnemonics to aid student’s memory
D. Link teaching and testing
 
9. After reading and paraphrasing Jose Rizal’s A la Juventud Filipina, Mr. Gomez asked
the class to share any insight derived from the poem. The class was asked to share
their insights about the poem. The ability to come up with an insight stems from the
ability to ________________
A. comprehend the subject that is being studied.
B. relate and organize things and ideas.
C. evaluate the worthiness of a thing.
D. analyze the parts of a whole.
 
10. Which learning principle marks the highlight of multiple intelligences?
A. Learning is static and permanent.
B. Intelligence is not measured in one form.
C. Cognitive theory is stressed in learning.
D. People have different IQ levels.

11. As a classroom manager, Miss Marianne is generous with sincere praise. What is
the advantage of her practice?
A. It reinforces positive behavior.
B. It is appreciated.
C. It conditions students’ discipline.
D. It satisfies students’ need for praise.
 
12. Teacher Hanna asked the students to describe how their families celebrate
holidays. Students can discover that people celebrate holidays differently. Which
principle in cognitive development governs Teacher Hanna’s teaching activity?
A. Social interaction is essential for cognitive development.
B. Children actively construct their knowledge.
C. Cognitive development involves relating new information to prior knowledge
D. Children often think in different ways at different ages.
 
13. Learner-centered assessments include which of the following?
A. only formative
B. summative and formative tests with feedback 
C. just feedback
D. lots of quizzes
 
14. Meaningfulness is the key to all cognitive processes. Meaningfulness depends on
_______.
A. The intensiveness of practice.
B. The persistence of students in answering.
C. The variety of learning activities. 
D. The manner in which the teacher handles the topic.
 
15. The process of problem solving and learning are highly unique and individual. This
principle means _________________.
A. each student becomes aware of how learning styles can be changed.
B. students can modify their own personal learning styles.
C. each student has his/her own distinctive style of learning and solving
problems. 
D. students can adapt alternative problem-solving models.
 
16. Friedrich Froebel believed that education should awaken the child's dormant
potential and stimulate them to the fullest development. In this case, the teacher's role is
that of a ________.
A. molder of character
B. facilitator of learning 
C. dispenser of knowledge
D. evaluator of learning
 
17. Mrs. Ramos provides reading materials of varying levels of difficulty to three groups
of learners. Which principle is implemented by her action?
A. Focus on the individual's need to achieve.
B. Help each student attain goals.
C. Focus on student attention.
D. Provide informative feedback.
 
18. Learning is incorporating and hierarchically organizing new material into one’s
cognitive structures. Which theory is explained?
A. Ausubel’s Subsumption Theory
B. Bandura’s Social Learning Theory
C. Bronfenbrenner’s Bioecological Theory
D. Piaget’s Cognitive Development Theory 
19. Adam strives hard to excel in oratory. Which would explain best his effort?
A. Desire to satisfy aesthetic needs.
B. Desire for security.
C. Desire for self-actualization.
D. Desire to know and understand.
  
20. The tendency to imitate elders is very strong in the early childhood stage. Teachers
should therefore be very good?
A. role models
B. counselors
C. facilitators of learning
D. disciplinarians
 
21. Visual imagery helps people store information in their memory more effectively.
What is one teaching implication of this principle?
A. Instruct students to take notes while you lecture.
B. Encourage your students to imagine the characters and situations when
reading a story.
C. Tell them to read more illustrated comics.
D. You will not object when students daydream in class.
 
22. Providing a variety of learning activities to students is a characteristic of a teacher
who understands the principle of:
A. Reward as a potential means of increasing the participation
B. Proactive teaching as a modern technique of teaching
C. Allowing students to be exposed to various teaching techniques.
 D. Facilitating learning with an emphasis on individual differences
 
23. Teacher Dennis begins a lesson on tumbling by demonstrating front and back
somersaults in slow motion and physically guiding his students through the correct
movements. As his students become more skillful, he stands back from the mat and
gives verbal feedback about how to improve. With Vygotsky’s theory in mind, what did
teacher Dennis do?
A. Scaffolding
B. Apprenticeship
C. Peer interaction
D. Guided participation
 
24. A reading teacher sees positive effects in the use of various sections of a
newspaper. In what way does the use of newspapers affect the students?
A. Provides for meeting pupils' different interests.
B. Strengthens learning skills.
C. Contributes to pupils' understanding.
D. Reinforces learning experiences.
 
25. Allowing students to participate in the formation of two to three rules for the class
promotes _____ in the classroom.
A. a sense of belongingness.
B. teacher diminished authority.
C. an autocratic atmosphere.
D. "buddy-buddy" system.
 
26. The use of mnemonics helps a pupil _______ information.
A. apply
B. understand
C. recite
D. remember
  
27. As a constructivist teacher, which of the following questions can be an interesting
one to tap the Grade 2 pupils' prior knowledge of the digestive system?
A. Why do you think our stomach hurts if we do not eat?
B. What is it about digestion that you learned before?
C. What are the organs responsible for digestion?
D. Do you know how the food we eat gets digested?

28. Psychologically, there is unity in all learning. However, there are distinct types of
learning that are recognized. One is affective learning that involves:
A. acquisition of attitudes ideals, judgment, and values. 
B. formation of concepts and ideas.
C. understanding of the external world through the use of the senses.
D. acquisition of facts and information.
29. Who of the following authors would most help teacher Lito to understand the
underlying effects of poverty on academic achievements.
A. Dewey
B. Kohlberg
C. Piaget
D. Maslow

30. A student has just learned a particular formula to solve a math problem. She tries it
with another problem and succeeds at it. So, this student is even more encouraged to
discover easier ways to solve math problems. Which of Thorndike's laws is depicted in
this situation?
A. Readiness
B. Effect 
C. Repetition
D. Practice
 
31. If a student is encouraged to develop himself to the fullest and must satisfy his
hierarchy of needs, the highest need according to Maslow is _______
A. belongingness.
B. self-actualization.
C. psychological.
D. safety needs.
 
32. When asked to do a learning task, Kian hesitates, “Mahirap. Ayaw ko. ‘Di ko kaya!”
(It’s difficult. I don’t like it. I can’t do it! Which statement about Kian is correct?
A. Wants to be sure of his self-efficacy
B. Has a high sense of self-efficacy
C. Has a low sense of self-efficacy
D. Has no sense of self-efficacy
 
33. What is the main role of the teacher?
A. Facilitator of Student learning 
B. Classroom manager
C. Model
D. Manager
 
34. Which of the following is a barrier to the improvement of memory?
A. Mnemonics
B. Retroactive inhibition 
C. Imagery
D. Word categorization
 
35. The 14 Learner-Centered psychological principles are divided along the following
areas, except _________.
A. physical and psychomotor
B. developmental and social
C. cognitive and metacognitive
D. individual differences
 
36. Which of the following situation best shows the teacher's consideration of the
learner's developmental patterns?
A. Teaching from the basic to the complex.
B. Using the best strategy for the topics.
C. Having outdoor activities for learning.
D. Developing the learners' hidden abilities.
 
37. The practice of motivating students before we proceed to our lesson proper is in
accordance with the Law of _______________.
A. Connectedness
B. Effect
C. Exercise
D. Readiness

38. After reading and paraphrasing Jose Rizal’s A la Juventud Filipina, Mr. Gomez
asked the class to share any insight derived from the poem. On which assumption
about the learner is Mr. Gomez’s act of asking the class to share their insight based?
A. Learners have multiple intelligences and varied learning styles.
B. Learners are producers of knowledge, not only passive recipients of
information.
C. Learners are like empty receptacles waiting to be filled up.
D. Learners are meant to interact with one another.
 
39. This educator opposes the traditional notion that students are empty vessels. He
calls this traditional technique the banking system.
A. Locke
B. Rousseau
C. Herbart
D. Freire 
 
40. To construct useful representations of knowledge and to acquire the thinking and
the learning strategies necessary for continued learning success across the life span,
students must generate and pursue personally relevant goals. This implies the teachers
must ___________________
A. group students according to type of goals.
B. share their personal goals with students.
C. segregate students without goals in order not to influence others.
D. show relevance of lesson to students’ goals.

41. The successful learner can link new information with existing knowledge in
meaningful ways. Who subscribes to such thought?
A. The cognitivist
B. The social-cognitivist
C. The constructivist
D. The behaviorist 
42. The audio-visual experience is an opportunity for the teacher to guide students for
more fruitful learning. It is more effective if students _______.
A. participate actively through discussion
B. are told to remember and memorize facts
C. listen attentively and jot down notes
D. are given quizzes immediately after the presentation

43. When a teacher attempts to develop his students' metacognitive skills, he teaches
them to:
A. Recall past lessons
B. Visualize
C. Formulate hypothesis
D. Think about their thinking

44. A child receives STARS STAMP after correctly completing his task. The child
always tries to complete all tasks correctly for him to have a stamp once again. What is
being shown in the situation?
A. Classical learning
B. Pavlovian conditioning
C. Operant learning
D. Associative learning
 
45. It includes everything from simple associations between stimuli and responses or
association learning to the development of complex insights as in problem-solving:
A. Affective learning
B. Cognitive learning
C. Psychomotor learning
D. All of these
46. Skinner’s operant conditioning model explains learning as a process
____________.
A. in which behavior is shaped and maintained by consequences.
B. of change based on insight and learning.
C. by which reflex action is modified and shaped.
D. of genetic shaping and change.
47. Mrs. Santos aims to maximize individual learning for her third-year students in
Science. If she intends to permit students to proceed at their own rate, which of the
following will she apply?
A. cooperative learning
B. modular instruction
C. hands-on learning
D. supervised study
48. SITUATION 
Paulo doesn’t enjoy writing but can’t escape from it. He has to finish it in order to
graduate. Miguel has always liked to write. He really likes to become a better
writer because she wants to be a journalist someday. Who is likely to be more
focused on the writing activity and why?
A. It cannot be determined. Motivation fluctuates.
B. Both, because they are both motivated anyway. 
C. Paulo, because he is extrinsically motivated.
D. Miguel, because he is intrinsically motivated.
 
49. Which is a classroom application of this principle, “Students learn more effectively
when they elaborate on new information.” Ask your students to ________________
A. write the principle five times.
B. commit the principle to memory.
C. analyze the principle from different points of view.
D. print the principle in bigger letters then put it in a place where they can read it
time and again.
 
50. A Music teacher is careful in preparing activities for each lesson. He praises liberally
and rewards correct answers. What view of learning is established?
A. Social learning
B. Operant conditioning
C. Classical conditioning
D. Meaningful learning
 
51. Jerome Brunner and Edgar Dale advise teachers to _______ for effective teaching-
learning.
A. dwell only on the abstract
B. move from the abstract to the concrete
C. dwell only on the concrete
D. move from the concrete to the abstract
52. Based on Bandura’s social learning theory, whom do children often imitate. Those
who ______.

I. have substantial influence over their lives

II. belong to their peer group

III. belong to other races

IV. are successful and seem admired


A. IV only
B. II and IV
C. I and IV
D. I and II
 
53. Which is a classroom application on operant conditioning?
A. Help students see the connectedness of facts, concepts, and principles.
B. Reinforce a good behavior to increase the likelihood that the learner will
repeat the response.
C. Create a classroom atmosphere that elicits relaxation
D. Make students learn by operating manipulatives.
 
54. Research says that children learn words best in contexts that are meaningful. What
does this imply to the teaching-learning process?
A. Reduce the number of new words taught for mastery.
B. Teach words in integrated contexts.
C. Retain out-of-context classroom drills.
D. Teach new words in isolation from other lessons for greater focus.

55. Repetition will facilitate the learning process when?


A. given the same information each time
B. kept short if possible
C. done in different setting
D. there are short intervals between repetitions
 
56. Under which generalization was the TV program "Batibot" developed?
A. Children learn by conditioning.
B. Children learn by discovery.
C. Children learn by trial and error.
D. Children learn by observing and imitating.
 
57. John Watson said: “Men are built, not born.” What does this statement point to?
A. The effect of environmental stimulation on a person’s environment
B. The absence of genetic influence on a person’s development
C. The ineffectiveness of training on a person’s development
D. The effect of heredity
 
58. Which of the following is usually considered the most important factor in a child’s
observable classroom behavior?
A. self-concept 
B. heredity
C. intelligence
D. cultural background
 
59. Which theory supports cooperative learning strategies?
A. Group games count a lot in assessing learning.
B. Learning is best when more interactions happen in a social setting.
C. There is no learning that happens in a vacuum.
D. Teachers must be a part of the group to facilitate learning.
 
60. Which of the following situation displays observational learning as contributed by
Bandura?
A. A child observing her mother put on a make-up and does not show any reaction
to it.
B. A child grabbing the make-up of her mother and using it as a toy.
C. A child imitating her mother putting on a make-up.
D. A child baby-talking to her mother and asking her why she wears make-up.
ED 8
Part 1
1. Which of the following statements has a very limited definition of educational
technology?
a. It is a profession composed of various job categories.
b. It refers to the computers used for teaching and learning.
c. It includes audiovisual materials, interactive multimedia and self-instructional
materials.
d. It is the development, application and evaluation of system, techniques and aids to
improve human learning
Answer: B
2. Which of the following statements is correct about the domains of educational
technology?
a. Design is the production stage while development is the planning stage.
b. Both the design and development are the planning stage.
c. Evaluation is synonymous with implementation.
d. Utilization is the action phase.
Answer: D
3. Ms. Gomez is planning to integrate technology in her Mathematics class. Which of
the following would be the logical steps in doing this?
I. Set the objectives
II. Analyze the learners
III. Utilize the materials with showmanship
IV. Evaluate the performance of the students
a. I, II, III, IV
b. II, I, III, IV
c. I, II, IV, III
d. II, I, IV, III
Answer: B
4. Which of the following is a limitation of models and real objects in teaching and
learning?
a. They pose problems on storage
b. They make learning more concrete.
c. They provide hands-on learning experiences.
d. They are readily available in the environment, around school and in the home.
Answer: A
5. Which group of technologies has the highest degree of concreteness?
a. Realia and computer
b. Video, picture and television
c. Digital video, film, versatile compact disc
d. Book, imaginative literature, programmed instruction
Answer: A
6. Mrs. Del Prado placed text together with the relevant graphics on the same page in
her multimedia presentation. Which principle did she apply?
a. Split attention
b. Spatial contiguity
c. Cost effectiveness
d. Communication effectiveness
Answer: A
7. Mrs. Olivarez presented real samples of rocks in her General Science class. What
principle did she apply?
a. Appropriateness
b. Authenticity
c. Responsiveness
d. Simplicity
Answer: B
8. Which is the best reason why teachers state the objectives before using instructional
media?
a. To secure materials
b. To prepare the materials beforehand.
c. To determine which media to use best.
d. To be able to practice how to operate the equipment
Answer: C
9. Which of the following should Mr. Rivera primarily consider in determining the
teaching-learning objectives and use of instructional media?
a. The assessment tool to be used
b. The learning activities
c. The learner
d. The teacher
Answer: B
10. Which of the following technologies provide iconic experiences to students/
children?
a. Video and books
b. Pictures and videos
c. Radio and recording
d. Modules and periodicals
Answer: B
11. Which of these technologies used in the classroom are arranged from the most
symbolic to multisensory?
a. Real objects, print, audio-visual materials and visual materials
b. Visual materials, audio visual materials, print and computers
c. Visual materials, print, audio-visual materials and realia
d. Print, audio-visual materials, computers and realia
Answer: D
12. Which of the following is inappropriate in using printed visuals such as charts,
graphs and drawings?
a. Provide written or verbal cues to highlight important aspect of visuals
b. Present the instructional materials simultaneously
c. Use materials that everyone can see
d. Make the presentation suspenseful
Answer: B
13. Susan wants to learn more English. Specifically, she wants to improve her listening
skills. She has a CD player, a tape recorder and has internet access. As an English
teacher, what do you suggest?
I. CDs with English listening drills
II. Tapes with English listening drills
III. Internet website such as Go4English, English Language Listening Lab or Randall's
listening Lab
a. I and II
b. II and III
c. I or III
d. I, II and III
Answer: D
14. Which of the following statements is incorrect about the contributions of technology
to student learning?
a. The quality of learning can be improved.
b. The delivery of instruction can be more interesting.
c. The method of teaching and learning becomes more interactive.
d. The role of the teacher can be changed into knowledge dispenser.
Answer: D
15. Mr. Tarnate, an ICT teacher takes into account technology standards to address the
needs of the students and help them adapt with the changing society and technology
Which of the following standards is an exception?
a. Creativity and innovation
b. Research and information literacy
c. Model digital-age work and learning
d. Technology operations and concepts
Answer: C
16. Ms. Vinluan, a computer teacher demonstrates understanding of local and global
issues and exhibits ethical and legal use of information and communications technology
tools. Which is true about her?
a. She models digital-age work and learning
b. She facilitates and inspires student learning and creativity.
c. She promotes and models digital citizenship and responsibility.
d. She designs and develops digital-age learning experiences and assessments
Answer: C
17. With the fast-paced evolution of technologies nowadays, why are teachers
encouraged to shift gradually from a teacher-centered instruction to a learner-centered
instruction?
I. A learner-centered instruction focuses on transformation of facts.
a. II and IV only
b. I, II and IV only
c. I, III and IV only
d. II, III and IV only
Answer: C
18. Ms. Hernandez employs student-centered instruction as the learners create their
digital portfolios in her computer class. What could be developed among them through
this approach?
a. Repetition and active learning
b. Mastery of skills and information delivery
c. Information processing and passive learning
d. Construction of knowledge and information exchange
Answer: D
19. Mr. Torres will have a multimedia presentation in his Science class. Which of the
following should he avoid?
a. Consider technical quality.
b. Apply different computer effects per slid.
c. Present information through graphic organizers
d. Use contrasting colors for text and background.
Answer: B
20. Mrs. Sison would like to integrate technology in writing a friendly letter. Which of the
following is the most effective way of doing it?
a. Let the pupils surf a friendly letter from the internet
b. Have the pupils write a friendly letter and send it through an email.
c. have the pupils forward a downloaded friendly letter to others via email.
d. Let the pupils write a friendly letter using word processing and have it critiqued by
their peers.
Answer: D
21. Which of the following computer-based instructional materials can be used to learn
new concepts?
a. Games
b. Tutorial
c. Simulation
d. Drill and practice
Answer: B
22. Professor dela Cruz would like to create a presentation material for her lesson on
the types of computer-assisted instruction. To make her presentation effective, which?
a. Situating tool
b. Informative tool
c. Productivity tool
d. Communicative tool
Answer: C
23. Professor Reyes is thinking of an online learning approach by which content
provides links to information at other locations and serves as a focal point for a distance
education experience. Which of the following should she use?
a. Teleconferencing
b. Self-paced program
c. Web-based instruction
d. Computer-aided instruction
Answer: C
24. Which is NOT a basic consideration in selecting and evaluating the content of an
educational technology tool?
a. Does it match the content?
b. Can it be easily dismantled?
c. Will it motivate and maintain interest?
d. Is there evidence of its effectiveness?
Answer: B
25. Your father wanted to finish his long dreamed course but he wanted to do it at home
during his free time. Would you recommend an online learning?
a. Yes, because online learning is the "in" thing
b. No, because online learning inhibits student-teacher interaction.
c. No, because hiring a helper would enable him to attend regularly in his class.
d. Yes, because he could learn at his own pace using a wide spectrum of technologies.
Answer: D
Part 2
1. Ms. Delos Santos is a fresh graduate teacher who was hired to teach in an
elementary school where there are enough resources for instruction. She wanted to
start preparing her instructional materials. Which is the most systematic process in
doing this?
a. Design, utilization, evaluation, development
b. Design, development, utilization, evaluation
c. Development, design, utilization, evaluation
d. Development, utilization, evaluation, design
Answer: B
2. Which is the most important reason why teachers need to preview their instructional
materials that will be used in the class?
a. To gain confidence in using them
b. To encourage viewers to be more focused
c. To avoid potential problems that might occur while materials are in use
d. To ensure appropriateness of the materials with the objectives and target audience
Answer: D
3. After Ms. Rivas planned her lesson in English, she found out that the materials at
hand do not match her objectives. Which is the best thing that she can do?
a. Modify the available materials
b. Teach the lesson the following day.
c. Change the objectives to match with available materials.
d. Carry out the lesson as planned and use the materials at hand.
Answer: A
4. Prof. Balagtas used worksheets, manipulatives, and models in teaching math to help
her students understand the lessons and love the subject. What did she bear in mind
when she used these materials?
a. Appropriateness
b. Balance
c. Breadth
d. Variety
Answer: D
5. With increasing use of educational technology inside the classroom, what role is
expected of the teacher?
a. Facilitator
b. Researcher
c. Knowledge giver
d. Source of information
Answer: A
6. It is impractical to bring real objects to the classroom so Aaron constructed a three-
dimensional visual instead. Which of the following did he construct?
a. Chart
b. Cartoon
c. Model
d. Graphic organizer
Answer: C
7. You asked your students to show a two-dimensional illustration of what they have
understood from what they have read. Which of the following non-projected visuals are
you referring to?
a. Graphic organizer
b. Print materials
c. Model
d. Realia
Answer: A
8. There are several reasons why teachers are reluctant in using electronic media in the
teaching-learning process. Which is the most common reason?
a. The limited exposure of teachers to new equipment
b. Their incompatibility to diverse needs of the learners
c. The difficulty in integrating technology in the curriculum
d. The excessive availability of local technology in the community
Answer: A
9. Prof. Mandanas would like to use an audio compact disc in teaching a lesson in
Filipino. In which activity in the teaching-learning process is it very effective?
a. In developing listening skills
b. In teaching creating thinking
c. In composing poems
d. In building concepts
Answer: A
10. Plants, pebbles, and blocks are just some of the effective instructional materials
readily found in the environment if they are utilized properly. Which of the following is
INCORRECT about their classroom use?
a. Pass a single object around the class.
b. Familiarize yourself with the object or model before its actual utilization.
c. Use the objects as springboard in encouraging students' active participation.
d. Make sure that the realia and model are large enough to be seen by the whole class.
Answer: A
11. Prof. Ruiz uses projected visuals such as Digital Liquid Projector (DLP) in
presenting her lesson. What could be her main reason for using it?
a. The projected materials are readily available.
b. They are more abstract than any other visuals.
c. Most projected visuals can be obtained at no cost.
d. She can easily prepare for own transparencies in advance.
Answer: A
12. Ms. Samonte used a film clip in teaching Social Studies concepts to her First Year
High School class. However, she found out that it was ineffectively used in the
classroom. When is technology considered INEFFECTIVE?
a. When it promotes mastery of the lesson
b. When it makes viewing more interesting
c. When it helps attain the objectives of the lesson
d. When it induces alienation on the part of the learners
Answer: D
13. Your principal purchased new computer units for your Learning Resource Center.
Which of the following should be your last consideration in using the technology?
a. Computers can be used for entertainment.
b. Computers can be used for research activity.
c. Computers can be used for interactive presentation.
d. Computers can be used to reinforce discussion of difficult concepts in class.
Answer: A
14. Computers can be classified according to the roles they play namely communicative
tool, informative tool, and constructive tool. What is the other role of computers not
mentioned in this item?
a. Instructional tool
b. Situating tool
c. Utility tool
d. Application tool
Answer: B
15. Which of the following categories of CAI will you use in your class if your objective is
to increase proficiency in a newly learned skill or refresh an existing one?
a. Tutorial
b. Simulation
c. Drill and practice
d. Instructional game
Answer: C
16. Which of the following is an ineffective use of Presentation software?
a. Darken the room.
b. Use appropriate pacing.
c. Read directly from the slides.
d. Allow interaction with the learner.
Answer: C
17. Which of the following is NOT an example of a communicative tool?
a. Chat
b. Electronic mail
c. Teleconferencing
d. Multimedia encyclopedia
Answer: D
18. Why is one-way delivery of information a misuse of communication tools?
a. Because the teacher expects the student to study more
b. Because it requires activities that focus on thinking than responding
c. Because it enables the users to focus more on higher level cognitive activities
d. Because this kind of practice lessens interaction capabilities of communication tools
Answer: D
19. Internet consists of thousands of connected computer networks around the world.
Which term does not refer to internet?
a. NET
b. On-line
c. Cyberspace
d. Information Superhighway
Answer: B
20. Which technology tool can Prof. Soriano use to communicate asynchronously with
her students?
a. Chat and blog
b. Chat and instant messaging
c. Blog and video conferencing
d. Electronic bulletin board and email
Answer: D
21. In your computer subject, you allow your class to chat as part of your motivation
before discussing to them the roles of computer as a tool. How is chat used in t his
context?
a. Information tool
b. Application tool
c. Communicative tool
d. Situating tool
Answer: B
22. Which statement is INCORRECT about computer conferencing?
a. It refers to live student interaction with an expert.
b. It is also known as discussion forum or bulletin board.
c. It also refers to online class discussions, forums or debates.
d. It permits two or more individuals
Answer: A
23. Which instructional application will you introduce to your class if your objective is to
help them find and use information resources available in the internet?
a. Webquests
b. Hybrid course
c. Scavenger Hunt
d. Distance education
Answer: C
24. In the delivery of distance education, what computer application is used to organize
instructions and track students records and progress?
a. Computer-based Multimedia
b. Computer-assisted Instruction
c. Computer-mediated Education
d. Computer-managed Instruction
Answer: B
25. When is distance education as effective as the traditional instruction?
a. When the method, technologist and assessment used are appropriate to the required
competencies.
b. When the course requires more face-to-face communication between the students
and teachers.
c. When students depend more on their online mentor.
d. When there is student to student interaction.
Answer: A
ED 9

1. When a school in the interrelation of what was taught, how it was taught, and what
was learned, this school believes in the curriculum view of ___________.
A. Arthur Bestor
B. John Dewey
C. Phillip Phenix
D. Hollis Caswell
 
2.  The best of feature of Tyler’s Rationale is the ____________.
A. evaluating the experiences
B. identifying the purpose
C. planning phase
D. organizing the experiences
 
3. It has been the practice of the teachers of Teresa Elementary School to conduct a
diagnostic test to learners during the first week of the class to measure what has
been learned by the learners in their previous grade levels and to pan for an
intervention to remedy the problems that were identified. This shows what type of
curriculum?
A. Written
B. Taught
C. Assessed
D. Supported
 
4. In a curriculum development class, the teacher asked the students to give an
enriched definition of the curriculum. Which among the following encompasses the
true essence of the term?
A. Curriculum is a list of courses in order to graduate.
B. Curriculum is a list of subjects to take to complete a course.
C. Curriculum is a never-ending process in education.
D. Curriculum is the sum total of all the learning experiences in the teaching-
learning process.
 
5. This curricularist believe that curriculum is about the plans of learning the teachers
are making to facilitate learning.
A. Tyler
B. Wheeler
C. Skilbeck
D. Taba 
 
6. The K to 12 Curriculum of the Department of Education in the Philippines follows
what view of the curriculum?
A. Traditional Point of View
B. Progressive Point of View
C. Linear Point of View
D. Spherical Point of View
 
7. This curricularist has published researches, books, manuals, and other instructional
materials. He/She is a/an __________.
A. planner
B. writer
C. evaluator
D. implementor
 
8. This curricularist believe that curriculum is about the experiences of the students
A. Tyler
B. Wheeler
C. Skilbeck
D. Taba
 
9. The Philippine Association for Teachers and Educators (PAFTE) proposed a new
curriculum for Teacher Education to make graduates globally competitive. What
type of curriculum is this?
A. Supported Curriculum
B. Hidden Curriculum
C. Assessed Curriculum
D. Recommended Curriculum
 
10.  Considering that curriculum is a dynamic process, this person put premium in the
role of the teacher’s participation in developing the curriculum:
A. Hilda Taba
B. Galen Saylor
C. Ralph Tyler
D. William Alexander
  
11.  The oldest and most conservative educational philosophy was
A. Essentialism
B. Reconstructionism
C. Progressivism
D. Perennialism

12. Teacher Agnes always values the importance of peer-tutoring inside her class, so
suggests that new and seasoned teachers should not neglect this kind of strategy
when teaching the learners? This shows what type of curriculum
A. Hidden Curriculum
B. Assessed Curriculum
C. Learned Curriculum
D. Taught Curriculum
 
13.  In order to have an effective teaching and learning, there must be an adequate
utilization of learning materials. What type of curriculum is this?
A. Assessed Curriculum
B. Supported Curriculum
C. Recommended Curriculum
D. Hidden Curriculum
 
14.  It is also known as the delivered curriculum.
A. Taught curriculum
B. Recommended curriculum
C. Written curriculum
D. Supported curriculum
 
15. Which of the following is NOT a characteristic of a good curriculum?
A. The curriculum is democratically conceived.
B. The curriculum is based on the needs of the people.
C. The curriculum is the result of an action plan.
D. The curriculum is continuously evolving.

16.  This curricularist attends seminars, workshops and pursues graduate work. He/She
isa/an __________.
A. knower
B. evaluator
C. innovator
D. writer
 
17.  Abraham Maslow became prominent in this theory.
A. Social Learning Theory
B. Classical Conditioning Theory
C. Hierarchy of Needs Theory
D. Meaningful Learning Theory
 
18.  Which of the following processes usually comes first in developing curriculum?
A. Organization of learning experiences
B. Selection of educational content
C. Selection of educational content
D. Evaluating educational experiences

19. These foundations show the chronological development of curriculum, mostly


shown using a timeline.
A. Sociological Foundations
B. Philosophical Foundations
C. Psychological Foundations
D. Historical Foundations
 
20. Who among the following believed that a curriculum follows a linear model which
means that the the school should plan and direct how learners should learn?
A. Wheeler
B. Tyler
C. Skilbeck
D. Oliva
 
21. This curricularist uses unique and out-of-the box strategies to make his/her class
highly engaging. He/She is a/an _______________.
A. innovator
B. knower
C. writer
D. implementer
 
22. Curriculum may be defined in many ways. What does this prove?
A. The concept of curriculum may be defined from different perspectives. 
B. The concept is limited and narrow inscope.
C. The curriculum is characterized asfragmentary, elusive, and confusing.
D. The concept of curriculum is based onthose given by experts.
 
23. What type of curriculum focuses on the changes that occurred in the learners due to
their school experiences.
A. Learned Curriculum
B. Taught Curriculum
C. Assessed Curriculum
D. Written Curriculum
 
24.  Which educational philosophy focuses on the importance of reforms and rebuilding
social and cultural infrastructure to improve society?
A. Reconstructionism
B. Progressivism
C. Essentialism
D. Perennialism
 
25. Which of the following views of curriculum focus on play, experience and interaction
of the pupils rather than a teacher-directed instruction?
A. Traditional Point of View
B. Progressive Point of View
C. Linear Point of View
D. Spherical Point of View
 
26. The Department of Education created several programs in the elementary and
secondary public schools to cater the different needs and skills of the learners,
some of these programs are Special Science Class (SSC), Special Program for the
Arts (SPA) and Special Program in Journalism (SPJ). These programs conform to
what type of curriculum?
A. Written
B. Taught
C. Assessed
D. Recommended
 
27. Ordinary people consider curriculum as ___________.

I. a list of subjects

II. courses to complete

III. subjects to undertake


A. I only
B. I, II, and III
C. II only
D. III only
 
28.  The curriculum in which the primary intention is to ensure that the educational
goals of the system are being accomplished is called
A. Supported curriculum
B. Written curriculum
C. Recommended curriculum
D. Taught curriculum
 
29.  The famous Russian physiologist, who introduced the theory of classical
conditioning was?
A. Ivan Pavlov 
B. Albert Bandura
C. Edward Thorndike
D. B.F. Skinner
 
30. This type of curriculum was based on the two principle, the General Leader and
Curriculum Leaders, where each has different sets of roles and responsibilities to
perform.
A. Behavioral Approach
B. Managerial Approach
C. System Approach
D. Humanistic Approach
 
31.  In analyzing the curriculum, the teacher should consider the following EXCEPT.

I. vision, mission, goals, and core values of the school

II. learning resources and faculties of the school

III. needs and interests of the learners

IV. all must be considered


A. II only
B. I, II, and III
C. III only
D. I only
 
32.  Which educational philosophy emphasizes the necessities of academic knowledge
and character development?
A. Progressivism
B. Perennialism
C. Reconstructionism
D. Essentialism
 
 
33. These curricularists developed the six approaches of curriculum which helped and
guide teachers and curriculum planners to craft effective and efficient curriculum
contents.
A. Campbell and Caswell
B. Ornsteins and Hunkins 
C. Tyler and Wheeler
D. Taba and Skilbeck
 
34.  When teachers conduct a series of evaluation to describe the extent of teaching,
what must be implemented?
A. Hidden Curriculum
B. Taught Curriculum
C. Learned Curriculum
D. Assessed Curriculum
 
35. Who proposed a spherical or cyclical view of curriculum?
A. Wheeler 
B. Tyler
C. Skilbeck
D. Oliva
36.  When curriculum is viewed as a process, what should the teacher consider
primarily in designing his/her lessons?
A. National competency-based competencies
B. Different modes of assessments
C. Various methods and strategies
D. Outcomes-based learning objectives
 
37. Mrs. Susan is teaching first graders, she found out that half of her class does not
know how to read. She implemented a reading intervention during her vacant
periods and after class hours. After three months of consistent remediation, all her
learners were able to read and even topped the reading assessment conducted by
the school. The approach that she used was ___________.
A. Managerial Approach
B. Behavioral Approach 
C. System Approach
D. Humanistic Approach
 
38.  What could be the content/topic when the teacher asked the learner to define
curriculum and complete a matrix on the differences between traditional and
progressive curriculum?
A. What could be the content/topic when the teacher asked the learner to define
curriculumand complete a matrix on the differences between traditional and
progressive curriculum?
B. Different elements that affect curriculum
C. The meaning of curriculum
D. Various curriculum perspectives
 
39.  All these questions should be answered by curriculum EXCEPT:
A. What subjects are important?
B. Who will benefit from a well-designed learning experience?
C. What qualifications should teachers have?
D. What outcomes should be achieved?
 
40. Teacher Jennifer believes that knowing how her learners performed in the test is
crucial in crafting an effective intervention project, so she always ensure to conduct
daily and weekly assessments of her learners. This shows that she is employing
what type of curriculum? 
A. Hidden Curriculum
B. Assessed Curriculum 
C. Learned Curriculum
D. Taught Curriculum
 
41. These curricularists defined curriculum as “a plan for providing sets of learning
opportunities for persons to be educated."
A. Campbell and Caswell
B. Ornsteins and Hunkins
C. J. G. Saylor, William M. Alexander, and Arthur J. Lewis 
D. Taba and Skilbeck
 
42.  The following are characteristics of progressive curriculum EXCEPT.
A. the classroom is only one place for the students to learn; learning can take place
anywhere. 
B. takes into account all the curricular elements and tests these elements through
real-life application.
C. knowledge that comes from various disciplines should be the focus.
D. focuses on the experiences of the students and supervised by the teacher. 
43. Which of the following best describes the etymology of curriculum?
A. Curriculum is a french word "cure"
B. Curriculum is a Latin word "currere"
C. Curriculum is German word "kureirin"
D. Curriculum is an English term for "care"
 
44. As a new teacher, Mrs. Romano is very eager to teach her learners based on what
was written on her Lesson Plan that she wrote herself? What role as a curricularist
does she portray?
A. Innovator
B. Planner
C. Writer
D. Implementer
 
45. Based on Oliva's definition of curriculum, which one best describes a classroom
teacher who makes use of the materials like, videos, stories, realia etc. in 
discussing her lesson?
A. Curriculum is that which is taught in school.
B. Curriculum is a program of studies
C. Curriculum is content
D. Curriculum is a set of materials
  
46.  He defined learning as habit formation.
A. Albert Bandura
B. Ivan Pavlov
C. B.F. Skinner
D. Edward Thorndike
 
47.  Mrs. Valdez asks her Grade 10 class to read biographies, encyclopedias, history of
books, and the like during their free time. She is a believer of what philosophy?
A. Essentialism
B. Behaviorism
C. Perennialism
D. Social Reconstructionism
 
48. Miss Lim started off her lesson plan with the outcomes she expects her students to
achieve at the end of the lesson. This curriculum approach is __________.

I. curriculum as a body of knowledge

II. curriculum as a process

III. curriculum as a product

IV. all of the above


A. II only
B. I only
C. I, II, and III
D. III only
 
 
49. During the onset of the COVID-19 pandemic, teachers crafted different instructional
materials such as, Self-Learning Modules, Learning Packets and Worksheets. This
shows what specific role of a teacher a curricularist?
A. Innovator
B. Planner
C. Writer
D. Implementer
 
50. Teacher Rod is teaching English 6, he is not a believer of the notion that there is no
size that fits all, that is why he varies his teaching styles and strategies in teaching
Parts of Speech to his learners. This show that he is following what type of
curriculum?
A. Written
B. Taught 
C. Assessed
D. Supported

You might also like